You are on page 1of 67

Target : JEE (Main) Conic Section

CONTENTS

 CONIC SECTION :

Topic Page No.

Theory –– 01 – 34

Exercise # 1 : Objective Questions –– 35 – 46

Exercise # 2 : Part – I : Objective Questions –– 46 – 50


: Part - II : Miscellaneous Questions

Exercise # 3 : JEE (Main) / AIEEE Problems –– 51 – 55


: Part – II :JEE (Advanced) / IIT-JEE Problems

Answers –– 56 – 57

Additional Problems for Self Practice (APSP) –– 58 –64


: Part - I : Practice Test Paper
: Part – II : Practice Questions

Answers –– – 65

JEE (Main) Syllabus

Parabola : Sections of cones, equation of parabola in standard form, condition for y = mx + c to be a


tangent and point (s) of tangency.
Ellipse : Equation of ellipse in standard form, condition for y = mx + c to be a tangent and point (s) of
tangency.
Hyperbola : Equation of hyperbola in standard form, condition for y = mx + c to be a tangent and point (s)
of tangency.
Conic Section

Our notion of symmetry is derived from the human face. Hence, we demand symmetry horizontally and in breadth only, not
vertically nor in depth .......... Pascal, Blaise
_____________________________________________________________________________________
_

This chapter focusses on parabolic curves, which constitutes one category of various curves obtained
by slicing a cone by a plane, called conic sections. A cone (not necessarily right circular) can be out in
various ways by a plane, and thus different types of conic sections are obtained. Let us start with the
definition of a conic section and then we will see how are they obtained by slicing a right circular cone.

1. Conic sections :
A conic section, or conic is the locus of a point which moves in a plane so that its distance from a fixed
point is in a constant ratio to its perpendicular distance from a fixed straight line.

(i) The fixed point is called the Focus.

(ii) The fixed straight line is called the Directrix.

(iii) The constant ratio is called the Eccentricity denoted by e.

PS
e
PM
(iv) The line passing through the focus & perpendicular to the directrix is called the Axis.

(v) A point of intersection of a conic with its axis is called a Vertex.

If S is (p, q) & directrix is x + my + n = 0


| x  my  n |
then PS = (x –  )2  (y – )2 & PM =
 2  m2
PS
=e  (2 + m2) [(x – p)2 + (y – q)2] = e2 (x + my + n)2
PM
Which is of the form ax2 + 2hxy + by2 + 2gx + 2fy + c = 0

2. Section of right circular cone by different planes :


A right circular cone is as shown in the figure – 1

A
Vertex
or
rat
ne

axis
Ge

B
Circular Base
Figure 1
Conic Section

(i) Section of a right circular cone by a plane passing through its vertex is a pair of straight lines
passing through the vertex as shown in the figure - 2.

Q
Figure –2
(ii) Section of a right circular cone by a plane parallel to its base is a circle as shown in the figure  3.

(iii) Section of a right circular cone by a plane parallel to a generator of the cone is a parabola as
shown in the figure–4.

Figure-4
(iv) Section of a right circular cone by a plane neither parallel to any generator of the cone nor
perpendicular or parallel to the axis of the cone is an ellipse or hyperbola as shown in the
figure  5 & 6.

Figure -5 Figure -6

3D View :
Conic Section

3. Distinguishing various conics :


The nature of the conic section depends upon the position of the focus S w.r.t. the directrix & also upon
the value of the eccentricity e. Two different cases arise.
(i) Case (I) When The Focus Lies On The Directrix.
In this case   abc + 2fgh  af2  bg2  ch2 = 0 & the general equation of a conic represents a pair of
straight lines if:
e > 1  h2 > ab the lines will be real & distinct intersecting at S.
e = 1  h2 > ab the lines will coincident.
e < 1  h2 < ab the lines will be imaginary.
(ii) Case (II) When The Focus Does Not Lie On Directrix.
a parabola an ellipse a hyperbola rectangular hyperbola

e = 1;   0, 0 < e < 1;   0; e > 1;   0; e > 1;   0

h² = ab h² < ab h² > ab h² > ab; a + b = 0

PARABOLA
1. Definition and terminology :
A parabola is the locus of a point, whose distance from a fixed point (focus) is equal to
perpendiculardistance from a fixed straight line (directrix).Four standard forms of the parabola are
y² = 4ax; y² =  4ax; x² = 4ay; x² =  4ay
For parabola y2 = 4ax:
Y

M P
L

N
A X
Z S

L'

P'
2
y = 4ax
(i) Vertex is (0, 0) (ii) focus is (a, 0)
(iii) Axis is y = 0 (iv) Directrix is x + a = 0
Focal Distance: The distance of a point on the parabola from the focus.
Focal Chord : A chord of the parabola, which passes through the focus.
Double Ordinate: A chord of the parabola perpendicular to the axis of the symmetry.
Latus Rectum: A double ordinate passing through the focus or a focal chord perpendicular to the axis
of parabola is called the Latus Rectum (L.R.).
For y² = 4ax.  Length of the latus rectum = 4a.
  ends of the latus rectum are L(a, 2a) & L’ (a,  2a).

NOTE :
(a) Perpendicular distance from focus on directrix = half the latus rectum.
(b) Vertex is middle point of the focus & the point of intersection of directrix & axis.
(c) Two parabolas are said to be equal if they have the same latus rectum.
Conic Section

Example # 1: Find the equation of the parabola whose focus is at (– 1, – 2) and the directrix is
x – 2y + 3 = 0.
Solution : Let P(x, y) be any point on the parabola whose focus is S(– 1, – 2) and the directrix
x – 2y + 3 = 0. Draw PM perpendicular to directrix x – 2y + 3 = 0. Then by definition,
SP = PM
 SP2 = PM2
2
 x  2y  3 
 (x + 1)2 + (y + 2)2 =  
 1 4 
 5 [(x + 1)2 + (y + 2)2] = (x – 2y + 3)2
 5(x2 + y2 + 2x + 4y + 5) = (x2 + 4y2 + 9 – 4xy + 6x – 12y)
 4x2 + y2 + 4xy + 4x + 32y + 16 = 0
This is the equation of the required parabola.

Example # 2 : Find the vertex, axis, focus, directrix, latusrectum of the parabola, also draw their rough
sketches. 4y2 + 12x – 20y + 67 = 0
Solution : The given equation is
67
4y2 + 12x – 20y + 67 = 0  y2 + 3x – 5y + =0
4
2 2
67 5 67  5 
 y2 – 5y = – 3x –  y2 – 5y +   = – 3x – + 
4 2 4 2
2 2
 5 42  5  7
  y   = – 3x –   y   = – 3 x   ....(i)
 2  4  2   2

7 5
Let x=X– ,y=Y+ ....(ii)
2 2
Using these relations, equation (i) reduces to
Y2 = – 3X ....(iii)
This is of the form Y2 = – 4aX. On comparing, we get 4a = 3
 a = 3/4.
Vertex - The coordinates of the vertex are (X = 0, Y = 0)
So, the coordinates of the vertex are
 7 5
 2, 2  [Putting X = 0, Y = 0 in (ii)]
 
Axis
The equation of the axis of the parabola is Y = 0.
So, the equation of the axis is
5
y= [Putting Y = 0 in (ii)]
2
Focus-
The coordinates of the focus are (X = –a, Y = 0)
i.e. (X = – 3/4, Y = 0).
So, the coordinates of the focus are
(–17/4, 5/2) [Putting X = 3/4 in (ii)]
Conic Section

Directrix -
3
The equation of the directrix is X = a i.e. X = .
4
So, the equation of the directrix is
11
x=– [Putting X = 3/4 in (ii)]
4
Latusrectum -
The length of the latusrectum of the given parabola is 4a = 3.

Self Practice Problems :


(1) Find the equation of the parabola whose focus is the point (0, 0)and whose directrix is the
straight line 3x – 4y + 2 = 0.
(2) Find the extremities of latus rectum of the parabola y = x 2 – 2x + 3.
(3) Find the latus rectum & equation of parabola whose vertex is origin & directrix is x + y = 2.
(4) Find the vertex, axis, focus, directrix, latusrectum of the parabola y2 – 8y – x + 19 = 0. Also
draw their roguht sketches.

1 9 3 9
Ans. (1) 16x2 + 9y2 + 24xy – 12x + 16y – 4 = 0 (2)  2, 4   2, 4 
   
(3) 4 2 , x2 + y2 – 2xy + 8x + 8y = 0
(4)

2. Parametric representation :
The simplest & the best form of representing the coordinates of a point on the parabola is (at², 2at) i.e.
the equations x = at² & y = 2at together represents the parabola y² = 4ax, t being the
parameterParametric form for :
y2 = – 4ax (–at2, 2at)
x = 4ay
2
(2at , at2)
x2 = – 4ay (2at , – at2)

Example # 3 : Find the parametric equation of the parabola (x – 1)2 = –16 (y – 2)


Solution :  4a = – 16  a = – 4, y – 2 = at2
x – 1 = 2 at  x = 1 – 8t, y = 2 – 4t2
Self Practice Problems :
(5) Find the parametric equation of the parabola x2 = 4ay
Ans. x = 2at, y = at2.

3. Position of a point relative to a parabola :


The point (x1 , y1) lies outside, on or inside the parabola y² = 4ax according as the expression y1²  4ax1
is positive, zero or negative.
S1 : y12 – 4ax1 Outside

S1 < 0  Inside Inside

P(x1, y1)
S1 > 0  Outside
Conic Section

Example # 4 : Check whether the point (3, 4) lies inside or outside the paabola y2 = 4x.
Solution : y2 – 4x = 0
 S1 y12 – 4x1 = 16 – 12 = 4 > 0
 (3, 4) lies outside the parabola.

Self Practice Problems :

(6) Find the set of value's of for which (, – 2 – ) lies inside the parabola y2 + 4x = 0.
Ans.   (– 4 – 2 3 , – 4 + 2 3 )

4. Line & a parabola :


The line y = mx + c meets the parabola y² = 4ax in two points real, coincident or imaginary according as
a  cm

Tangent
Secant
A

 condition of tangency is, c = a/m.


Length of the chord intercepted by the parabola
 4 
on the line y = m x + c is :.  2 a(1  m2 )(a  mc)
m 
NOTE :
(i) The equation of a chord joining t1 & t2 is 2x  (t1 + t2) y + 2 at1 t2 = 0.

(ii) If t1 & t2 are the ends of a focal chord of the parabola y² = 4ax then t1t2 = 1. Hence the

a 2a 
coordinates at the extremities of a focal chord can be taken as (at², 2at) &  2 , 
t t 
Focal chord
A

S (focus)
B

(iii) Length of the focal chord making an angle  with the x axis is 4acosec² 

Example # 5 : Discuss the position of line y = x + 1 with respect to parabola y2 = 4x.


Solution : Solving we get (x + 1)2 = 4x  (x – 1)2 = 0
so y = x + 1 is tangent to the parabola.

Example # 6 : Prove that focal distance of a point P(at2,2at) on parabola y2=4ax(a>0)is a(1+ t2).
Solution :

 PS = PM = a + at2  PS = a (1 + t2).
Conic Section

Example # 7 : If the endpoint t1, t2 of a chord of the parabola y2 = x satisfy the relation t1 t2 = 8 then prove that
the chord always passes through a fixed point. Find the point?
1 1  1 1 
Solution : Equation of chord joining  t12 , t1  and  t 22 , t 2  is
4 2  4 2 
1 2  1  1 2 1 1
y– t1 =  x  t12   (t1 + t2) y – t1 – t1t2 = 2x – t12
2 t1  t 2  4  2 2 2
2
y= (x + 2) (t1t2 = 8)
t1  t 2
 This line passes through a fixed point (– 2, 0).

a
Example # 8 : Prove that the straight line y = mx + c touches the parabola y2 = 4a (x + a) if c = ma +
m
Solution : Equation of tangent of slope ‘m’ to the parabola y2 = 4a(x + a) is
a  1
y = m(x + a) +  y = mx + a  m  
m  m 
but the given tangent is y = mx +c
a
c = am +
m
Self Practice Problems :

(7) If the line y = 3x + intersect the parabola y2 = 4x at two distinct point's then set of value's of ''
is

(8) Find the midpoint of the chord x + y = 2 of the parabola y2 = 4x.

(9) If one end of focal chord of parabola y2 = 16x is (16, 16) then coordinate of other end is.

(10) If PSQ is focal chord of parabola y2 = 4ax (a > 0), where S is focus then prove that
1 1 1
+ = .
PS SQ a

(11) Find the length of focal chord whose one end point is ‘t’.
2
 1
Ans. (7) (– , 1/3) (8) (4, – 2) (9) (1, – 4) (11) at  
 t
5. Tangents to the parabola y² = 4ax :
Equation of tangent at a point on the parabola can be obtained by replacement method or using
derivatives.
In replacement method, following changes are made to the second degree equation to obtain T.
x2  x x1, y2  y y1, 2xy  xy1 + x1y, 2x  x + x1, 2y  y + y1
So, it follows that the targents are :

(i) y y1 = 2 a (x + x1) at the point (x1, y1) ;


a  a 2a 
(ii) y = mx + (m  0) at  2 ,
m m m 
(iii) t y = x + a t² at (at², 2at).

Note : Point of intersection of the tangents at the point t1 & t2 is { at1 t2 , a(t1 + t2) }.
Conic Section

Example # 9 : A tangent to the parabola y2 = 8x makes an angle of 45° with the straight line y = 3x + 5. Find
its equation and its point of contact.
3 1
Solution : Slope of required tangent’s are m =
1 3
1
m1 = – 2, m2 =
2
a
Equation of tangent of slope m to the parabola y2 = 4ax is y = mx + .
m
1 
tangent’s y = – 2x – 1 at  ,  2 
 2 
1
y= x + 4 at (8, 8)
2

Example # 10 : Find the equation to the tangents to the parabola y2 = 9x which goes through the point (4, 10).
9
Solution : Equation of tangent to parabola y2 = 9x is y = mx +
4m
Since it passes through (4, 10)
9 1 9
 10 = 4m +  16 m2 – 40 m + 9 = 0  m= ,
4m 4 4
x 9
 equation of tangent’s are y= +9 & y = x + 1.
4 4

Example # 11 : Find the equations to the common tangents of the parabolas y2 = 4ax and x2 = 4by.
Solution : Equation of tangent to y2 = 4ax is
a
y = mx + ........(i)
m
Equation of tangent to x2 = 4by is
b 1 b
x = m 1y +  y= x– ........(ii)
m1 m1 (m1 )2
for common tangent, (i) & (ii) must represent same line.
1 a b
 =m & = – 2
m1 m m1
1/ 3
a  a
 = – bm2  m = 
m  b
1/ 3 1/ 3
 a  b
 equation of common tangent is y =    x + a 
 b  a
Self Practice Problems :
(12) Find equation tangent to parabola y2 = 4x whose intercept on y–axis is 2.
(13) Prove that perpendicular drawn from focus upon any tangent of a parabola lies on the tangent
at the vertex.
(14) Prove that image of focus in any tangent to parabola lies on its directrix.
(15) Prove that the area of triangle formed by three tangents to the parabola y2 = 4ax is half the area
of triangle formed by their points of contacts.
x
Ans. (12) y  2
2
Conic Section

6. Normals to the parabola y² = 4ax :


Normal is obtained using the slope of tangent. P

Normal
2a y
Slope of tangent at (x1 , y1) =  Slope of normal = – 1
y1 2a
y1
(i) y  y1 = – (x  x1) at (x1, y1) ;
2a
(ii) y = mx  2am  am3 at (am2,  2am)
(iii) y + tx = 2at + at3 at (at2, 2at).
NOTE :
(a) Point of intersection of normals at t1 & t2 is (a (t 12 + t 22 + t1t2 + 2),  a t1 t2 (t1 + t2)).
(b) If the normals to the parabola y² = 4ax at the point t1, meets the parabola again at
 2
the point t2, then t2 = –.  t1  
 t1 
P(t1)

Q(t2)

(c) If the normals to the parabola y² = 4ax at the points t1 & t2 intersect again on the parabola at the
point 't3' then t1 t2 = 2; t3 =  (t1 + t2) and the line joining t1 & t2 passes through a fixed point (2a, 0).

(d) If normal are drawn from a point P(h, k) to the parabola y2 = 4ax then
k = mh  2am  am3 i.e. am3 + m(2a  h) + k = 0.
2a  h k
m1 + m 2 + m 3 = 0 ; m1m2 + m2m3 + m3m1 = ; m1 m2 m 3 =  .
a a
Where m1, m2, & m3 are the slopes of the three concurrent normals. Note that
A

P(h, k)

B
C
A, B, C  Conormal points
 algebraic sum of the slopes of the three concurrent normals is zero.
 algebraic sum of the ordinates of the three conormal points on the parabola is zero
 Centroid of the  formed by three conormal points lies on the xaxis.
Example # 12 : Find the locus of the point N from which 3 normals are drawn to the parabola y2 = 4ax are such
that
(i) Two of them are equally inclined to x-axis
(ii) Two of them are perpendicular to each other
Solution : Equation of normal to y2 = 4ax is
y = mx – 2am – am3
Let the normal passes through N(h, k)
k = mh – 2am – am3  am3 + (2a – h) m + k = 0
For given value’s of (h, k) it is cubic in ‘m’.
Let m1, m2 & m3 are root’s of above equation
 m1 + m 2 + m 3 = 0 ......(i)
2a  h
m1m2 + m2m3 + m3m1 = ......(ii)
a
k
m1m2m3 = – ......(iii)
a
Conic Section

(i) If two normal are equally inclined to x-axis, then m1 + m2 = 0


 m3 = 0  y=0
(ii) If two normal’s are perpendicular
 m1 m2 = – 1
k
from (3) m3 = .....(iv)
a
k 2a  h
from (2) –1+ (m1 + m2) = .....(v)
a a
k
from (1) m1 + m 2 = – .....(vi)
a
from (5) & (6), we get
k2 h
–1– =2–
a a
y = a(x – 3a)
2

Self Practice Problems :

(16) Find the points of the parabola y2 = 4ax at which the normal is inclined at 30° to the axis.

(17) If the normal at point P(1, 2) on the parabola y2 = 4x cuts it again at point Q then Q = ?

(18) Find the length of normal chord at point ‘t’ to the parabola y2 = 4ax.

(19) If normal chord at a point 't' on the parabola y2 = 4ax subtends a right angle at the vertex then
prove that t2 = 2

(20) Prove that the chord of the parabola y2=4ax, whose equation is y–x 2 +4a 2 = 0, is a
normal to the curve and that its length is 6 3a .

(21) If the normals at 3 points P, Q & R are concurrent, then show that
(i) The sum of slopes of normals is zero, (ii) Sum of ordinates of points P, Q, R is zero
(iii) The centroid of PQR lies on the axis of parabola.
3
a 2a   a 2a  4a(t 2  1) 2
Ans. (16)  , ,  ,  (17) (9, – 6) (18)  
 3 3  3 3  t2
7. Pair of tangents :
The equation to the pair of tangents which can be drawn from any point (x 1, y1) to the parabola y² = 4ax
is given by: SS1 = T² where :
S  y²  4ax ; S1 = y1²  4ax1 ; T  y y1  2a(x + x1).

A
(x1, y1) P
B

Example # 13 : Write the equation of pair of tangents to the parabola y2 = 4x drawn from a point P(–1, 2)
Solution : We know the equation of pair of tangents are given by SS1 = T²
 (y2 – 4x) (4 + 4) = (2y – 2 (x – 1))2
 8y2 – 32x = 4y2 + 4x2 + 4 – 8xy + 8y – 8x
 y2 – x2 + 2xy – 6x – 2y = 1
Conic Section

Example # 14 : Find the directrix of parabola such that from point P tangents are drawn to parabola
y2 = 4ax having slopes m1, m2 such that m1 m2 = –2
Solution : Equation of tangent to y2 = 4ax, is
a
y = mx +
m
Let it passes through P(h, k)
 m2h – mk + a = 0
m1 m2 = –2  a = –2
so directrix is x = 2

Self Practice Problems :

(22) If two tangents to the parabola y2 = 4ax from a point P make angles 1 and 2 with the axis of
the parabola, then find the locus of P in each of the following cases.
(i) tan21 + tan22 =  (a constant) (ii) cos 1 cos 2 =  (a constant)
Ans. (i) y2 – 2ax = x2 , (ii) x2 = 2 {(x – a)2 + y2}

8. Director circle :
Locus of the point of intersection of the perpendicular tangents to a curve is called the Director Circle.
For parabola y2 = 4ax it’s equation is x + a = 0 which is parabola’s own directrix.

9. Chord of contact :
Equation to the chord of contact of tangents drawn from a point P(x1,y1) is yy1=2a(x+x1).
A

P
(x1, y1 ) Chord of contact

Note : The area of the triangle formed by the tangents from the point (x1, y1) & the chord of
1
contact is (y ²  4ax1)3/2
2a 1

Example # 15: If the line x – y – 5 = 0 intersect the parabola y2 = 8x at P & Q, then find the point of intersection
of tangents at P & Q.
Solution : Let (h, k) be point of intersection of tangents then chord of contact is
yk = 4(x + h)
4x – yk + 4h = 0 .....(i)
But given is x – y – 5 = 0
3 k 4h
 = =  h = – 5, k = 4  point (–5, 4)
2 1 5

Self Practice Problems :


(23) Prove that locus of a point whose chord of contact w.r.t. parabola passes through focus is
directrix
(24) If from a variable point ‘P’ on the line x – 2y + 1 = 0 pair of tangent’s are drawn to the parabola
y2 = 8x then prove that chord of contact passes through a fixed point, also find that point.
Ans. (25) (1, 8)
Conic Section

10. Chord with a given middle point :


Equation of the chord of the parabola y² = 4ax whose middle point is

M(x,y)

2a
(x1, y1) is y  y1 = (x  x1)  T = S1
y1

Example # 16 : Find the locus of middle point of the chord of the parabola y2 = 4ax which pass through a given
point (p, q).
Solution : Let P(h, k) be the mid point of chord of parabola y2 = 4ax,
so equation of chord is yk – 2a(x + h) = k2 – 4ah.
Since it passes through (p, q)
 qk – 2a (p + h) = k2 – 4ah
 Required locus is
y2 – 2ax – qy + 2ap = 0.

Example # 17 : Find the locus of middle point of the chord of the parabola y2 = 4ax which are parallel to the line
x +2y = 0
Solution : Let P(h, k) be the mid point of chord of parabola y2 = 4ax,
so equation of chord is yk – 2a(x + h) = k2 – 4ah.
2a k 2a
but slope = =m =
k 2 1
locus is k = –4a y = 4a

Self Practice Problems :

(25) Find the equation of chord of parabola y2 = 4x whose mid point is (4, 2).

(26) Find the locus of mid - point of chord of parabola y2 = 4ax which touches the parabola x2 = 4by.

Ans. (25) x–y–2=0 (26) y (2ax – y2) = 4a2b

11. Important Highlights :


(i) If the tangent & normal at any point ‘P’ of the parabola intersect the axis at T & G then
ST = SG = SP where ‘S’ is the focus. In other words the tangent and the normal at a point P on
the parabola are the bisectors of the angle between the focal radius SP & the perpendicular
from P on the directrix. From this we conclude that all rays emanating from S will become
parallel to the axis of theparabola after reflection.

(ii) The portion of a tangent to a parabola cut off between the directrix & the curve subtends a right
angle at the focus. See figure above.
Conic Section

(iii) The tangents at the extremities of a focal chord intersect at right angles on the directrix, and
hence a circle on any focal chord as diameter touches the directrix. Also a circle on any focal
radii of a point P (at2, 2at) as diameter touches the tangent at the vertex.
P

R
S A
O

(iv) Any tangent to a parabola & the perpendicular on it from the focus meet on the tangent at the
vertex.
y
P
R

x
O S

(v) Semi latus rectum of the parabola y² = 4ax, is the harmonic mean between segments of any
focal chord of the parabola.
P

2(PS)(SQ)
S = 2a
Q
PS  SQ

(vi) The area of the triangle formed by three points on a parabola is twice the area of the triangle
formed by the tangents at these points.

ELLIPSE
In this chapter we are going to discuss in detail the nature of path in which on planets move around the
sun. They follow on elliptical path with the sun at one of its foci. Let us look at the definition of ellipse.
1. Definitions :
It is locus of a point which moves in such a way that the ratio of its distance from a fixed point and a
fixed line (not passes through fixed point and all points and line lies in same plane) is constant (e),
which is less than one.
The fixed point is called - focus
The fixed line is called -directrix.
The constant ratio is called - eccentricity, it is denoted by 'e'.
Example # 18 : Find the equation to the ellipse whose focus is the point (– 1, 1), whose directrix is the straight
1
line x – y + 3 = 0 and eccentricity is .
2
2
PS 1 1 hk 3
Solution : Let P  (h, k) be moving point, e = =  (h + 1)2 + (k – 1)2 =  
PM 2 4 2 

 locus of P(h, k) is 8 {x2 + y2 + 2x – 2y + 2} = (x2 + y2 – 2xy + 6x – 6y + 9)


7x2 + 7y2 + 2xy + 10x – 10 y + 7 = 0.
Note : The general equation of a conic with focus (p, q) & directrix x + my + n = 0 is:
(2 + m2) [(x  p)2 + (y  q)2] = e2 (x + my + n)2  ax2 + 2hxy + by2 + 2gx + 2fy + c = 0
represent ellipse if 0 < e < 1;   0, h² < ab
Conic Section

Self Practice Problems :


1
(27) Find the equation to the ellipse whose focus is (0, 0) directrix is x + y – 1 = 0 and e = .
2
Ans. 3x2 + 3y2 – 2xy + 2x + 2y – 1 = 0.

2. Standard equation :
Standard equation of an ellipse referred to its principal axes along the coordinate axes is
x2 y2
 = 1, where a > b & b² = a² (1  e²).
a2 b2

b2
(i) Eccentricity: e = 1  , (0 < e < 1)
a2

(ii) Focii : S  (a e, 0) & S ( a e, 0).

a a
(iii) Equations of Directrices : x = &x= .
e e

(iv) Major Axis : The line segment AA in which the focii S & S lie is of length 2a & is called the major
axis (a > b) of the ellipse. Point of intersection of major axis with directrix is called the foot of the
directrix (Z).

(v) Minor Axis : The yaxis intersects the ellipse in the points B  (0,  b) & B  (0, b). The line
segment BB is of length 2b (b < a) is called the minor axis of the ellipse.

(vi) Principal Axis : The major & minor axes together are called principal axis of the ellipse.

(vii) Vertices : Point of intersection of ellipse with major axis. A ( a, 0) & A  (a, 0) .

(viii) Focal Chord : A chord which passes through a focus is called a focal chord.

(ix) Double Ordinate : A chord perpendicular to the major axis is called a double ordinate.

(x) Latus Rectum : The focal chord perpendicular to the major axis is called the latus rectum.
2
2b2  minor axis 
Length of latus rectum (LL) =
a

major axis

 2a 1  e2 
= 2 e (distance from focus to the corresponding directrix)
Centre : The point which bisects every chord of the conic drawn through it, is called the centre of the
y2
conic. C  (0, 0) the origin is the centre of the ellipse x2  2 = 1.
2

a b
Conic Section

y2
If the equation of the ellipse is given as x2 
2
Note : (i) = 1 and nothing is mentioned, then
a b2
the rule is to assume that a > b.
(ii) If b > a is given, then the yaxis will become major axis and x-axis will become the minor axis
and all other points and lines will change accordingly.

x2 y2
Equation : 2
+ =1
a b2
b
Foci (0, be) Directrices : y = 
e
a2
a2 = b2 (1 – e2), a < b.  e= 1–
b2
Vertices (0, b) ; .R. y = be
2
2a
 (L·R.) = , centre : (0, 0)
b
Example # 19 : Find the equation to the ellipse whose centre is origin, axes are the axes of co-ordinate and
passes through the points (2, 2) and (3, 1).
x2 y2
Solution : Let the equation to the ellipse is 2 + 2 = 1
a b
Since it passes through the points (2, 2) and (3, 1)
4 4
 + =1 ..........(i)
a2 b2
9 1
and + =1 .........(ii)
a2 b2
from (i) – 4 (ii), we get
4  36 32
2
= 1– 4  a2 =
a 3
from (i), we get
1 1 3 83 32
2
= – =  b2 =  Ellipse is 3x2 + 5y2 = 32
b 4 32 32 5
1
Example # 20 : Find the equation of the ellipse whose focii are (4, 0) and (– 4, 0) and eccentricity is
3
Solution : Since both focus lies on x-axis, therefore x-axis is major axis and mid point of focii is origin
which is centre and a line perpendicular to major axis and passes through centre is minor axis
which is y-axis.
x2 y2
Let equation of ellipse is 2 + 2 = 1
a b
1
 ae = 4 and e= (Given)
3
 a = 12 and b2 = a2 (1 – e2)
 1
 b2 = 144  1    b2 = 16 × 8  b=8 2
 9
x2 y2
Equation of ellipse is + =1
144 128
Conic Section

Example # 21 : If lenght of latus rectum is equal to half of semi major axis then find the eccentricity of ellipse
x2 y2
Solution : Let the equation of ellipse is 2
+ =1 (a > b)
a b2

2b2 a b2 1 b2 3
=  =  e = 1 2 =
a 2 a2 4 a 2

Example # 22 : Find the equation of axes, directrix, co-ordinate of focii, centre, vertices, length of latus-rectum
(x  2)2 (y  4)2
and eccentricity of an ellipse + =1.
25 16

X2 Y2
Solution : Let x – 2 = X, y – 4 = Y, so equation of ellipse becomes as + =1.
52 42
equation of major axis is Y = 0  y = 4.
equation of minor axis is X = 0  x = 2.
centre (X = 0, Y = 0)  x = 2, y = 4
C  (2, 4)
Length of semi-major axis a = 5
Length of major axis 2a = 10
Length of semi-minor axis b = 4
Length of minor axis = 2b = 8.
Let 'e' be eccentricity
 b2 = a2 (1 – e2)

a2  b2 25  16 3
e= 2
= = .
a 25 5

2b2 2  16 32
Length of latus rectum = LL = = =
a 5 5
Co-ordinates focii are X = ± ae, Y = 0
 S  (X = 3, Y = 0) & S (X = –3, Y = 0)
 S  (5, 4) & S (–1, 4)

Co-ordinate of vertices

Extremities of major axis A  (X = a, Y = 0) & A (X = – a, Y = 0)


 A  (x = 7, y = 4) & A = (x = – 3, y = 4)
A  (7, 4) & A (– 3, 4)
Extremities of minor axis B (X = 0, Y = b) & B  (X = 0, Y = – b)
B (x = 2, y = 8) & B (x = 2, y = – 0)
B  (2, 8) & B (2, 0)
a
Equation of directrix X = ±
e
25 31 19
x–2=±  x= & x=–
3 3 3
Conic Section

Self Practice Problems :

(28) Find the equation to the ellipse whose axes are of lengths 6 and 2 6 and their equations are
x – 3y + 3 = 0 and 3x + y – 1 = 0 respectively.

(29) Find the eccentricity of ellipse whose minor axis is double the latus rectum.

(30) Find the co-ordinates of the focii of the ellipse 4x2 + 9y2 = 1.

x2 y2 1
(31) Find the standard ellipse 2
+ = 1 passing through (2, 1) and having eccentricity
a b2 2

Ans. (28) 3(x – 3y + 3)2 + 2(3x+ y – 1)2 = 180,


21x2 – 6xy + 29y2 + 6x – 58y – 151 = 0.

3  5 
(29) (30)   , 0 (31) 3x2 + 4y2 = 16
2  6 
 

3. Auxiliary circle / eccentric angle :


A circle described on major axis of ellipse as diameter is called the auxiliary circle.
Let Q be a point on the auxiliary circle x² + y² = a² such that line through Q perpendicular to the x  axis
on the way intersects the ellipse at P, then P & Q are called as the Corresponding Points on the
ellipse & the auxiliary circle respectively. ‘’ is called the Eccentric Angle of the point P on the ellipse
( < ).
Q  (a cos , a sin)
P  (a cos , b sin)

Focal property : summation of distance of any point from two focus is equal to lenght of major axis
Proof : Let 'e' be the eccentricity of ellipse.
 PS = e . PM
a 
= e   a cos  
e 
PS = (a – a e cos)
 a
and PS = e. PM= e  acos   
 e
PS = a + ae cos
 focal distance are (a ± ae cos)
PS + PS = 2a, PS + PS = AA= lenght of major axis
Conic Section

Self Practice Problems :


x2 y2
(32) Find the distance from centre of the point P on the ellipse + = 1 whose eccentric angle
a2 b2
is 
x2 y2
(33) Find the eccentric angle of a point on the ellipse + = 1 whose distance from the
6 2
centre is 2.
 3
Ans. (32) r  a2 cos2   b2 sin2  (33) ± ,±
4 4

4. Parametric representation :
y2
The equations x = a cos  & y = b sin  together represent the ellipse x2  2 = 1.
2

a b
Where  is a parameter. Note that if P()  (a cos  b sin ) is on the ellipse then;
Q()  (a cos  a sin ) is on the auxiliary circle.
The equation to the chord of the ellipse joining two points with eccentric angles  &  is given by .
x  y  
cos  sin  cos
a 2 b 2 2

x 2 y2    5 
Example # 23 : Write the equation of chord of an ellipse + =1 joining two points P   and Q   .
25 16  4  4 
  5  y   5    5 
x 4 4  4 4    
Solution : Equation of chord is cos   + 4 . sin   = cos  4 4 
5 2 2 2
x  3  y  3  x y
. cos   + . sin   = 0 – + = 0  4x = 5y
5  4  4  4  5 4

Self Practice Problems :


(34) Show that the sum of squares of reciprocals of two perpendicular diameters of the ellipse
x2 y2
+ = 1 is constant. Find the constant also.
a2 b2
1 1 1 
Ans.  2
 2
4a b 

5. Position of a point w.r.t. an ellipse :


x12 y12
The point P(x1, y1) lies outside, inside or on the ellipse according as ;   1 > < or = 0.
a2 b2
x 2 y2
Example # 24: Check whether the point P(5, 4) lies inside or outside of the ellipse + =1.
25 16
25 16
Solution : S1  + –1=1+1–1>0
25 16
 Point P  (5, 4) lies outside the ellipse.

6. Line and an ellipse :


x2 y2
The line y = mx + c meets the ellipse 2
+ = 1 in two points real, coincident or imaginary
a b2
according as c² is < = or > a²m² + b².
x2 y2
Hence y = mx + c is tangent to the ellipse + = 1 if c² = a²m² + b².
a2 b2
Conic Section

Example # 25 : Find the set of value(s) of '' for which the line 3x – 4y +  = 0 lies outside the ellipse
x2 y2
+ = 1.
16 9
(4y  )2 y2
Solution : Solving given line with ellipse, we get + =1
9  16 9
2y 2 y 2
– + –1 = 0
9 18 144
Since, line intersect the parabola at two distinct points,
 roots of above equation are real & distinct
 D0
2 8  2 
 – .  1 < 0    (–, –12 2 )  (12 2 , )
(18)2 
9  144 

Self Practice Problems :


x2 y2
(35) Find the value of '' for which 2x – y +  = 0 touches the ellipse + =1
25 9
Ans.  = ± 109

7. Tangents :
x2 y2
(i) Slope form: y = mx ± a 2 m2  b 2 is tangent to the ellipse + =1 for all values of m.
a2 b2
x x1 y y1 x2 y2
(ii) Point form :   1 is tangent to the ellipse + = 1 at (x1, y1).
a2 b2 a2 b2
x cos  y sin  x2 y2
(iii) Parametric form:   1 is tangent to the ellipse 2 + 2 = 1 at the point
a b a b
(a cos , b sin ).

Note :
(a) There are two tangents to the ellipse having the same m, i.e. there are two tangents parallel to
any given direction.These tangents touches the ellipse at extremities of a diameter.
 cos     
2
sin 2
(b) Point of intersection of the tangents at the point  &  is,  a 
,b 
 cos
 2
cos  2  

(c) The eccentric angles of the points of contact of two parallel tangents differ by .

Example # 26 : Find the equations of the tangents to the ellipse 3x2 + 4y2 = 12 which are perpendicular to the
line x + y = 3.
Solution : Slope of tangent = m = 1
x2 y2
Given ellipse is + =1
4 3

Equation of tangent whose slope is 'm' is y = mx ± 4m2  3


 m=1
 y=x± 7
Conic Section

x2 y2
Example # 27 : A tangent to the ellipse +
= 1 meets the co-ordinate axes at A and B respectively. If P
2
a b2
divides AB in the ratio 3 : 1, then find the locus of point P.
Solution : Let P  ( a cos, b sin)
 equation of tangent is
x y
cos + sin = 1
a b

A  (a sec, 0)
B  (0, b cosec)
 P divide AB internally in the ratio 3 : 1
a 3a
h= :k=
4cos 4 sin
a 3a
 cos = , sin =
4h 4k
a2 9a2
then the locus is  =1
16x 2 16y 2
Self Practice Problems :
(36) Show that the locus of the point of intersection of the tangents at the extremities of any focal
chord of an ellipse is the directrix corresponding to the focus.
(37) Show that the locus of the foot of the perpendicular on a varying tangent to an ellipse from
either of its foci is a concentric circle.

(38) Prove that the portion of the tangent to an ellipse intercepted between the ellipse and the
directrix subtends a right angle at the corresponding focus.

(39) Find the area of parallelogram formed by tangents at the extremities of latera recta of the
x2 y2
ellipse   1.
a2 b2
2a3
Ans. (39)
a2  b2
8. Normals :
x2 y2 a2 x b2 y
(i) Equation of the normal at (x1, y1) to the ellipse + = 1 is  = a²  b².
a2 b2 x1 y1

x2 y2
(ii) Equation of the normal at the point (acos , bsin ) to the ellipse + = 1 is;
a2 b2
ax. sec   by. cosec  = (a²  b²).

(iii) Equation of a normal in terms of its slope 'm' is y = mx 


a 2

 b2 m
.
a 2  b 2 m2
Conic Section

x2 y2
Example # 28 : P and Q are corresponding points on the ellipse +
= 1 and the auxiliary circles
2
a b2
respectively. The normal at P to the ellipse meets CQ in R, where C is the centre of the ellipse.
Prove that CR = a + b
Solution :

Let P  (acos , b sin)


 Q  (a cos, a sin)
Equation of normal at P is
(a sec) x – (b cosec ) y = a2 – b2 ..........(i)
equation of CQ is y = tan . x .........(ii)
Solving equation (i) & (ii), we get (a – b) x = (a – b2) cos
2

x = (a + b) cos, & y = (a + b) sin


R  ((a + b) cos, (a + b) sin)  CR = a + b

Self Practice Problems :


x2 y2
(40) Find the value(s) of 'k' for which the line x + y = k is a normal to the ellipse  1
a2 b2
2

Ans. k= 
a 2
 b2 
a2  b2

9. Pair of tangents :
The equation to the pair of tangents which can be drawn from any point (x1, y1) to the ellipse
x2 y2
 = 1 is given by: SS1 = T² where :
a2 b2
x2 y2 x12 y12 xx1 yy1
S  –1 ; S1 =  –1; T 2
+ –1
a 2
b 2
a 2
b2
a b2

x2 y2
Example # 29 : How many real tangents can be drawn from the point (4, 3) to the ellipse + = 1. Find
16 9
the equation of these tangents & angle between them.
Solution : Given point P  (4, 3)
x2 y2
ellipse S  + –1=0
16 9
16 9
S1  + –1=1>0  Point P  (4, 3) lies outside the ellipse.
16 9
Two tangents can be drawn from the point P(4, 3).
Equation of pair of tangents is SS1 = T2
2
 x2 y 2   4x 3y  x2 y2 x2 y2 xy x 2y
    1 . 1 =    1  + –1= + +1+ – –
 16 9   16 9  16 9 16 9 6 2 3
 
 – xy + 3x + 4y – 12 = 0  (4 – x) (y – 3) = 0  x=4&y=3

and angle between them =
2
Conic Section

Alternative

By direct observation
x = 4, y = 3 are tangents.
x2 y2
Example # 30 : Find the locus of point of intersection of perpendicular tangents to the ellipse  =1
a2 b2
Solution : Let P(h, k) be the point of intersection of two perpendicular tangents
equation of pair of tangents is SS1 = T2
2
 x2 y 2   h2 k 2   hx ky 
  2  2  1  2  2  1 =  2  2  1
a b  a b  a b 

x2  k 2  y2  h2 
   1 +   1 + ........ = 0 .........(i)
a2  b2 
 b2  a2 
Since equation (i) represents two perpendicular lines
1  k2  1  h2 
 
2  2
 1 + 2  2  1 = 0
a b  b  a 
 
 k –b +h –a =0
2 2 2 2
 locus is x2 + y2 = a2 + b2
Self Practice Problems :
(41) Find the locus of point of intersection of the tangents drawn at the extremities of a focal chord
x2 y2
of the ellipse + = 1.
a2 b2
a
Ans. x=±
e
10. Director circle :
Locus of the point of intersection of the tangents which meet at right angles is called the Director
Circle. The equation to this locus is x² + y² = a² + b² i.e. a circle whose centre is the centre of the ellipse
& whose radius is the length of the line joining the ends of the major & minor axes.

Example # 31: An ellipse slides between two perpendicular lines. Show that the locus of its centre is a circle.
Solution : Let length of semi-major and semi-minor axis are 'a' and 'b' and centre is C(h, k)
Since ellipse slides between two perpendicular lines, therefore point of intersection of two
perpendicular tangents lies on director circle.
Let us consider two perpendicular lines as x & y axes
 point of intersection is origin O  (0, 0)
 OC = radius of director circle

 h2  k 2 = a2  b2
 locus of C  (h, k) is  x2 + y2 = a2 + b2 which is a circle
Self Practice Problems :
(42) A tangent to the ellipse x2 + 4y2 = 4 meets the ellipse x2 + 2y2 = 6 at P and Q. Prove that the
tangents at P and Q of the ellipse x2 + 2y2 = 6 are at right angles.
Conic Section

11. Chord of contact :


x2 y2
Equation to the chord of contact of tangents drawn from a point P(x1, y1) to the ellipse + = 1 is
a2 b2
xx1 yy1
T = 0, where T= 2
+ –1
a b2
Self Practice Problems :
(43) Find the locus of point of intersection of tangents at the extremities of normal chords of the
x2 y2
ellipse 2
+ = 1.
a b2
(44) Find the locus of point of intersection of tangents at the extremities of the chords of the ellipse
x2 y2
+ = 1 subtending a right angle at its centre.
a2 b2
a6 b6 x2 y2 1 1
Ans. (43) 2
+ 2
= (a2 – b2)2 (44) + = 2
+
x y a 4
b 4
a b2
12. Chord with a given middle point :
y2
Equation of the chord of the ellipse x2  2 = 1 whose middle point is (x1, y1) is T = S1,
2

a b
2 2
x1 y1 xx1 yy1
where S1 =  –1; T 2
+ – 1.
a 2
b2
a b2
Self Practice Problems :
x2 y2
(45) Find the equation of the chord + = 1 which is bisected at (2, 1).
36 9
Ans. x + 2y = 4

13. Important highlights :


x2 y2
Refering to the ellipse  =1
a2 b2

(i) If P be any point on the ellipse with S & S as its foci then (SP) + (SP) = 2a.

(ii) The tangent & normal at a point P on the ellipse bisect the external & internal angles between
the focal distances of P. This refers to the well known reflection property of the ellipse which
states that rays from one focus are reflected through other focus & viceversa. Hence we can
deduce that the straight
lines joining each focus to the foot of the perpendicular from the other focus upon the tangent at
any point P meet on the normal PG and bisects it where G is the point where normal at P
meets the major axis.
Conic Section

(ii) The product of the length’s of the perpendicular segments from the foci on any tangent to the
ellipse is b² and the feet of these perpendiculars lie on its auxiliary circle.

(iv) The portion of the tangent to an ellipse between the point of contact & the directrix subtends a
right angle at the corresponding focus.

(v) If the normal at any point P on the ellipse with centre C meet the major & minor axes in G & g
respectively, & if CF be perpendicular upon this normal, then
(a) PF. PG = b² (b) PF. Pg = a²
(c) locus of the mid point of Gg is another ellipse having
the same eccentricity as that of the original ellipse.

[where S and S are the focii of the ellipse]

(vi) The circle on any focal distance as diameter touches the auxiliary circle.

HYPERBOLA
Hyperbolic curves are of special importance in the field of science and technology especially astronomy
and space studies. In this chapter we are going to study the characteristics of such curves.

1. Definition :
A hyperbola is defined as the locus of a point moving in a plane in such a way that the ratio of its
distance from a fixed point to that from a fixed line (the point does not lie on the line) is
a fixed constant greater than 1.
PS
= e >1, e – eccentricity
PM
Conic Section

2. Standard equation & definition(s)

x2 y 2
Standard equation of hyperbola is –  1 , where b2 = a2 (e2  1).
a2 b2

b2
(i) Eccentricity (e) : e2 = 1 +
a2
(ii) Foci : S  (ae, 0) & S  ( ae, 0).
a a
(iii) Equations of directrices : x = & x=
e e
(iv) Transverse axis : The line segment AA of length 2a in which the foci S & S both lie is called
the transverse axis of the hyperbola.

(v) Conjugate axis : The line segment BB of length 2b between the two points
B (0,  b) & B (0, b) is called as the conjugate axis of the hyperbola.

(vi) Principal axes : The transverse & conjugate axis together are called principal axes of the
hyperbola.

(vii) Vertices : A (a, 0) & A ( a, 0)

(viii) Focal chord : A chord which passes through a focus is called a focal chord.

(ix) Double ordinate : A chord perpendicular to the transverse axis is called a double ordinate.

(x) Latus rectum : Focal chord perpendicular to the transverse axis is called latus rectum.
Its length () is given by
2
2b2  C.A.
=  = 2a (e2  1)
a T. A.
Note : (a) Length of latus rectum = 2 e × (distance of focus from corresponding directrix)
 b2   b2   b2   b2 
(b) End points of latus rectum are L   ae,  , L   ae,   , M   ae,  , M   ae,  
 a a a a
       
(xi) Centre: The point which bisects every chord of the conic, drawn through it, is called the centre
x2 y2
of the conic. C  (0,0) the origin is the centre of the hyperbola  1 .
a2 b2
(xii) General note : Since the fundamental equation to hyperbola only differs from that to ellipse in
having b2 instead of b2 it will be found that many propositions for hyperbola are derived from
those for ellipse by simply changing the sign of b2.
Conic Section

Example # 32 : Find the equation of the hyperbola whose directrix is 3x + 4y = 2, focus (1, 2) and
eccentricity 3 .
Solution :

Let P(x,y) be any point on the hyperbola.


Draw PM perpendicular from P on the directrix.
Then by definition SP = e PM
 (SP)2 = e2 (PM)2
2
 3x  4y  2 
 (x – 1)2 + (y – 2)2 = 3  
 5 
which is the required hyperbola.

Example # 33: Find the eccentricity of the hyperbola whose latus rectum is half of its transverse axis.
x2 y2
Solution: Let the equation of hyperbola be – = 1.
a2 b2
2b2 2b2 1
Then transverse axis = 2a and latus–rectum = . According to question = (2a)
a a 2
 2b2 = a2 ( b2 = a2 (e2 – 1))
3
 2a2 (e2 – 1) = a2  2e2 – 2 = 1  e2 =
2
3 3
 e= Hence the required eccentricity is .
2 2
3. Conjugate hyperbola :
Two hyperbolas such that transverse & conjugate axes of one hyperbola are respectively the conjugate
& the transverse axes of the other are called conjugate hyperbolas of each other.

x2 y2 x2 y2
eg.  1 &    1 are conjugate hyperbolas of each other.
a2 b2 a2 b2

y2 x2
Equation : 2
– =1
b a2
a2
a2 = b2 (e2 – 1)  e= 1
b2
2a2
Vertices(0,  b) ;  (L.R.) =
b
Conic Section

Note :
(a) If e1 & e2 are the eccentrcities of the hyperbola & its conjugate then e12 + e22 = 1.
(b) The foci of a hyperbola and its conjugate are concyclic and form the vertices of a
square.

(c) Two hyperbolas are said to be similar if they have the same eccentricity.

(d) Two similar hyperbolas are said to be equal if they have same latus rectum.

(e) If a hyperbola is equilateral then the conjugate hyperbola is also equilateral.

Example # 34 : Find the lengths of transverse axis and conjugate axis, eccentricity, the co-ordinates of foci,
vertices, length of the latus-rectum and equations of the directrices of the following hyperbola
16x2 – 9y2 = – 144.
x2 y2
Solution : The equation 16x2 – 9y2 = –144 can be written as – =–1
9 16

x2 y2
This is of the form – =–1
a2 b2
 a2 = 9, b2 = 16  a = 3, b = 4

Length of transverse axis : The length of transverse axis = 2b = 8

Length of conjugate axis : The length of conjugate axis = 2a = 6

 a2   9  5
Eccentricity : e =  1  2  =  1  16  =
 b    4

Foci : The co-ordinates of the foci are (0, + be) i.e., (0, + 5)

Vertices : The co–ordinates of the vertices are (0, + b) i.e., (0, + 4)


2a2 2(3)2 9
Length of latus–rectum : The length of latus–rectum = = =
b 4 2
Equation of directrices : The equation of directrices are
b 4 16
y=+ y = +  y=+
e (5 / 4) 5
Self Practice Problems :

(46) Find the equation of the hyperbola whose foci are (6, 4) and (– 4, 4) and eccentricity is 2.

(47) Obtain the equation of a hyperbola with coordinates axes as principal axes given that the
distances of one of its vertices from the foci are 9 and 1 units.
x 2 y2
(48) The foci of a hyperbola coincide with the foci of the ellipse + = 1. Find the equation of
25 9
the hyperbola if its eccentricity is 2.
x2 y2 y2 x2
Ans. (46) 12x2 – 4y2 – 24x + 32y – 127 = 0 (47) – = 1, – =1
16 9 16 9
(48) 3x2 – y2 – 12 = 0.
Conic Section

4. Auxiliary circle :
A circle drawn with centre C and transverse axis as a diameter is called the auxiliary circle of the
hyperbola. Equation of the auxiliary circle is x2 + y2 = a2.
Note from the following figure that P & Q are called the "corresponding points" of the hyperbola & the
auxiliary circle.

5. Parametric representation :
x2 y2
The equations x = a sec & y = b tan  together represent the hyperbola   1 where is a
a2 b2
parameter.
Note that if P()  (a sec  b tan ) is on the hyperbola then,
Q()  (a cos  a sin ) is on the auxiliary circle.
The equation to the chord of the hyperbola joining the two points P( & Q( is given by
x  y    
. cos  sin  cos
a 2 b 2 2

6. Position of a point 'P' w.r.t. a hyperbola :


2 2
x1 y1
The quantity S1    1 is positive, zero or negative according as the point (x 1, y1) lies inside, on
a2 b2
or outside the curve.

Example # 35 : Find the position of the point (5, – 4) relative to the hyperbola 9x2 – y2 = 1.
Solution : Since 9 (5)2 – (–4)2 – 1 = 225 – 16 – 1 = 208 > 0,
So the point (5,–4) lies inside the hyperbola 9x2 – y2 = 1..
7. Line and a hyperbola :
x2 y2
The straight line y = mx + c is a secant, a tangent or passes outside the hyperbola  1
a2 b2
according as : c2 > or = or < a2 m2  b2, respectively.
8. Tangents :
x2 y2
(i) Slope form : y = m x ± a2m – b2 can be taken as the tangent to the hyperbola 2
  1,
a b2
having slope 'm'.
x2 y2
(ii) Point form: Equation of tangent to the hyperbola   1 at the point (x1, y1) is
a2 b2
xx1 yy1
 2 1
a2 b
x2 y2
(iii) Parametric form: Equation of the tangent to the hyperbola   1 at the point. (a sec ,
a2 b2
x sec  y tan 
b tan ) is  1
a b
Conic Section

 1  2 
 cos 2    2 
Note : (a) Point of intersection of the tangents at P(1) & Q(2) is  a , b tan  1 

 cos 1   2  2 

 2 

(b) If |1 + 2| = , then tangents at these points (1 & 2) are parallel.

(c) There are two parallel tangents having the same slope m. These tangent touches the
hyperbola at the extremities of a diameter.

x2 y2
Example # 36 : For what value of  for which line 2x + y +  = 0 touches the hyperbola – =1
16 9
x2 y2
Solution : This line will touch the hyperbola – = 1 if c2 = a2m2 – b2
a2 b2
2 = 16 × 4 – 9
2= 55  = 55

Self Practice problems :


(49) For what value of  does the line y = 2x + touches the hyperbola 16x2 – 9y2 = 144 ?

(50) Find the equation of the tangent to the hyperbola x2 – y2 = 1 which is parallel to the line
4y = 5x + 7.
Ans. (49) =±2 5 (50) 4y = 5x ± 3

9. Normals :
x2 y2
(i) The equation of the normal to the hyperbola   1 at the point P (x1, y1) on it is
a2 b2
a2 x b2 y
 = a2 + b2 = a2 e2.
x1 y1

x2 y2
(ii) The equation of the normal at the point P (a sec , b tan ) on the hyperbola   1 is
a2 b2
ax by
 = a2 + b2 = a2 e2.
sec  tan 
(a2  b2 )m
(iii) Equation of normals in terms of its slope 'm' are y = mx 
a 2 – b 2m 2
Self Practice problems :
x2 y2
(51) Prove that the line lx + my – n = 0 will be a normal to the hyperbola – =1
a2 b2
a2 b2 (a2  b2 )2
if – = .
2 m2 n2
(52) Find the locus of the foot of perpendicular from the centre upon any normal to the hyperbola
x2 y2
– = 1.
a2 b2

Ans. (52) (x2 + y2)2 (a2y2 – b2x2) = x2y2 (a2 + b2)2


Conic Section

10. Pair of tangents :


The equation to the pair of tangents which can be drawn from any point (x1, y1) to the hyperbola

x2  y = 1 is given by: SS = T² where :


2

a2 b2 1

y2 x12 y12 xx1 yy1


S  x2  2 – 1 ;
2
S1 =  –1; T – – 1.
a b a 2
b 2
a 2
b2

x2 y2
Example # 37 : How many real tangents can be drawn from the point (4, 3) to the hyperbola – =1.
16 9
Find the equation of these tangents & angle between them.
Solution : Given point P  (4, 3)
x2 y2
Hyperbola S – –1=0
16 9
16 9
 S1  – –1=–1<0
16 9
 Point P  (4, 3) lies outside the hyperbola.
 Two tangents can be drawn from the point P(4, 3).
Equation of pair of tangents is SS1 = T2
2
 x2 y2   4x 3y 
 .    1 (– 1) =    1
 16 9   16 9 
 

x2 y2 x2 y2 xy x 2y
 – + +1= + +1– – +
16 9 16 9 6 2 3
 4
 3x2 – 4xy – 12x + 16y = 0  = tan–1  
3
11. Director circle :
The locus of the point of intersection of the tangents which are at right angle is known as the director
circle of the hyperbola. The equation to the director circle is : x2 + y2 = a2  b2.
If b2 < a2 , then the director circle is real.
If b2 = a2 (i.e. rectangular hyperbola), then the radius of the director circle is zero and it reduces to a
point circle at the origin. In this case centre is the only point from which two perpendicular tangents can
be drawn on the curve.
If b2 > a2, then the radius of the director circle is imaginary, so that there is no such circle and so no pair
of tangents at right angle can be drawn to the curve.

12. Chord of contact :

Equation to the chord of contact of tangents drawn from a point P(x 1, y1) to the hyperbola

x2 y2 xx1 yy1
– = 1 is T = 0, where T= 2
– –1
a 2
b2
a b2
Conic Section

x2 y2
Example # 38: If tangents to the parabola y2 = 4ax intersect the hyperbola 2
– = 1 at A and B, then find
a b2
the locus of point of intersection of tangents at A and B.

Solution: Let P  (h, k) be the point of intersection of tangents at A & B


xh yk
Equation of chord of contact AB is 2
– =1 ......(i)
a b2
Which touches the parabola
Equation of tangent to parabola y2 = 4ax
a a
y = mx +  mx – y = – ......(ii)
m m
equation (i) & (ii) as must be same
a

m 1 m h b2 ak
 = = m = 2
&m=– 2
 h   k  1 k a b
 2 
 2
a   b 

hb2 ak b4
 =– locus of P is y2 = – .x
ka2 b2 a3

13. Chord with a given middle point :


y2
Equation of the chord of the hyperbola x2  2 = 1 whose middle point is (x1, y1) is T = S1, where
2

a b
x12 y12 xx1 yy1
S1 =  –1; T 2
– – 1.
a 2
b 2
a b2

x2 y2
Example # 39 : Find the locus of the mid - point of chords of the hyperbola – = 1.
25 9
Solution : Let P  (h, k) be the mid-point
xh yk h2 k2
 equation of chord whose mid-point is given is – –1= – –1
a2 b2 a2 b2

6x 4y x2 y2
– = – –1
25 9 25 9

Self Practice Problems :


x2 y2
(53) Find the equation of the chord – = 1 which is bisected at (2, 1).
36 9

(54) Find the point 'P' from which pair of tangents PA & PB are drawn to the hyperbola
x2 y2
– = 1 in such a way that (5, 2) bisect AB
25 16
 20 8 
Ans. (53) x = 2y (54)  , 
 3 3
Conic Section

14. Rectangular hyperbola (equilateral hyperbola) :


The particular kind of hyperbola in which the lengths of the transverse & conjugate axis are equal is
called an Equilateral Hyperbola. Note that the eccentricity of the rectangular hyperbola is 2 .
Since a = b
b2
equation becomes x2 – y2 = a2  e= 1 = 1 1 = 2
a2

Rotation of this system through an angle of 45° in clockwise direction gives another form to the
equation of rectangular hyperbola.
a2
which is xy = c2 where c2 = .
2
Rectangular hyperbola (xy = c2) :
Vertices : (c, c) & ( c,  c);
Foci :   
2 c, 2 c &  2 c, 2 c , 
Directrices : x + y =  2c
Latus Rectum (l ) :  = 2 2 c = T.A. = C.A.
Parametric equation x = ct, y = c/t, t  R – {0}
Equation of a chord joining the points P (t1) & Q(t2) is x + t1 t2 y = c (t1 + t2).
 x y  x
Equation of the tangent at P (x1, y1) is    = 2 & at P (t) is + t y = 2 c.
 x1 y1  t
Equation of the normal at P (t) is x t  y t = c (t  1).
3 4

Chord with a given middle point as (h, k) is kx + hy = 2hk.


Example # 40 : A triangle has its vertices on a rectangular hyperbola. Prove that the orthocentre of the triangle
also lies on the same hyperbola.
Solution : Let "t1", "t2" and "t3" are the vertices of the triangle ABC, described on the rectangular
hyperbola xy = c2.
 c  c 
Co–ordinates of A,B and C are  ct1,  ,  ct 2 , 
 t1   t2 
 c 
and  ct 3 ,  respectively
 t3 

c(t 3  t 2 ) 1
Now slope of BC is =–
c (t 2  t 3 ) t 2 t 3 t2 t3
 Slope of AD is t2t3
 Slope of AD is t2t3
c
Equation of Altitude AD is y – = t2t3(x – ct1)
t1
or t1y – c = x t1t2t3 – ct12t2t3 .....(1)
Similarly equation of altitude BE is
t2y – c = x t1t2t3 – ct1t22t3 ......(2)
 c 
Solving (1) and (2), we get the orthocentre   , ct1t 2 t 3  which lies on xy = c2.
 t1t 2 t 3 
Conic Section

15. Important results :

(i) Difference of focal distances is a constant, i.e. |PS – PS| = 2a


x2 y2
(ii) Locus of the feet of the perpendicular drawn from focus of the hyperbola   1 upon any
a2 b2
tangent is its auxiliary circle i.e. x2 + y2 = a2 & the product of these perpendiculars is b2.

P
T
S S

T

(iii) The portion of the tangent between the point of contact & the directrix subtends a right angle at
the corresponding focus.

S
T

(iv) The tangent & normal at any point of a hyperbola bisect the angle between the focal radii. This
explains the reflection property of the hyperbola as "An incoming light ray " aimed towards
one focus is reflected from the outer surface of the hyperbola towards the other focus. It follows
that if an ellipse and a hyperbola have the same foci, they cut at right angles at any of their
common point.

x2 y2 x2 y2
(v) Note that the ellipse   1& the hyperbola  = 1 (a > k > b > 0) are
a2 b2 a2  k 2 k 2  b2
confocal and therefore orthogonal.

A rectangular hyperbola circumscribing a triangle also passes through the orthocentre of this
 
triangle. If  c ti , c  i = 1, 2, 3 be the angular points P, Q, R then orthocentre is
 t i

 c 
 t t t ,  c t1 t 2 t 3  .
 1 2 3 
Conic Section

Example # 41 : A ray originating from the point (5, 0) is incident on the hyperbola 9x 2 – 16y2 = 144 at the point
P with abscissa 8. Find the equation of the reflected ray after first reflection and point P lying
in first quadrant.
x2 y2
Solution : Given hyperbola is 9x2 – 16y2 = 144. This equation can be rewritten as – = 1 ....(1)
16 9
Since x co–ordinate of P is 8. Let y
co–ordinate of P is .
 (8,) lies on (1)

64  2
 – = 1  2 = 27   = 3 3 ( P lies in first quadrant)
16 9
Hence co-ordinate of point P is (8,3 3 ).
 Equation of reflected ray passes through P (8,3 3 ) and S(–5,0)

03 3
 Its equation is y–3 3 = (x – 8) or 13y – 39 3 = 3 3 x – 24 3
5  8
or 3 3 x – 13y + 15 3 = 0.
Conic Section

 Marked Questions may have for Revision Questions.


OBJECTIVE QUESTIONS
PARABOLA
Section (A) : Elementary Concepts of Parabola

A-1. The equation of the parabola whose focus is ( 3, 0) and the directrix is, x + 5 = 0 is:
(1) y2 = 4 (x  4) (2) y2 = 2 (x + 4) (3) y2 = 4 (x  3) (4) y2 = 4 (x + 4)

A-2. If (2, 0) is the vertex & y  axis is the directrix of a parabola, then its focus is:
(1) (2, 0) (2) ( 2, 0) (3) (4, 0) (4) ( 4, 0)

A-3. Length of the latus rectum of the parabola 25 [(x  2)2 + (y  3)2] = (3x  4y + 7)2 is:
(1) 4 (2) 2 (3) 1/5 (4) 2/5
2
A-4. The point on the parabola y = 12x whose focal distance is 4, are
  
(1) 2, 3 , 2,  3   
(2) 1, 2 3 , 1,  2 3 
(3) (1, 2) (4) none of these

A-5. The latus rectum of a parabola whose directrix is x + y – 2 = 0 and focus is (3, – 4), is
(1) –3 2 (2) 3 2 (3) 2 2 (4) 3/ 2

A-6. A parabola is drawn with its focus at (3, 4) and vertex at the focus of the parabola y2  12 x  4 y + 4 =0.
The equation of the parabola is:
(1) x2  6 x  8 y + 25 = 0
2
(2) y  8 x  6 y + 25 = 0
(3) x2  6 x + 8 y  25 = 0 (4) x2 + 6 x  8 y  25 = 0

A-7. The length of the side of an equilateral triangle inscribed in the parabola, y2 = 4x so that one of its
angular point is at the vertex is:
(1) 8 3 (2) 6 3 (3) 4 3 (4) 2 3

2
A-8. The ends of latus rectum of parabola x + 8y = 0 are
(1) (–4, –2) and (4, 2) (2) (4, 2) and (–4, 2) (3) (–4, –2) and (4, –2) (4) (4, 2) and (4, –2)

A-9. The equation of the latus rectum of the parabola x2 + 4x + 2y = 0 is


(1) 2y + 3 = 0 (2) 3y = 2 (3) 2y = 3 (4) 3y = – 2
2
A-10. Vertex of the parabola 9x – 6x + 36y + 9 = 0 is
 1 2  1 1  1 1  1 1
(1)  ,–  (2)  – ,–  (3)  – ,  (4)  , 
3 9  3 2  3 2 3 2

A-11.The focus of the parabola is (1, 1) and the tangent at the vertex has the equation x + y = 1. Then:
(1) length of the latus rectum is 2 2
2
(2) equation of the parabola is (x  y) = 4 (x + y  1)
(3) the co-ordinates of the vertex are (1/2, 1/2)
(4) All of these
Conic Section

The focal distance of a point on the parabola y2 = 16 x whose ordinate is twice the abscissa, is
A-12.
(1) 6 (2) 8 (3) 10 (4) 12

A-13. Which one of the following equations parametrically represents equation to a parabolic profile?
t
(1) x = 3 cos t; y = 4 sin t (2) x2  2 =  2 cos t; y = 4 cos2
2
t t
(3) x = tan t; y = sec t (4) x = 1  sin t ; y = sin + cos
2 2

A-14.
The latus rectum of a parabola whose focal chord is PSQ such that SP = 3 and SQ = 2 is given by:
(1) 24/5 (2)) 12/5 (3) 6/5 (4) none of these

Section (B) : Position of point /Line, Chord

B-1. Point (2,3) lies


(1) In side the parabola y2 = 4x (2) In side the parabola x2 = 4y
2
(3) On the parabola y = 4x (4) On the parabola x2 = 4y

B-2. If the point (– 1,) lies in side the parabola x2 = 4(y–1) then range of values of  is
(1) (1, 5) (2) (4, 7) (3) (2, 9) (4) (0, 4)

B-3. The length of chord intercepted on the line 2x + y = 2 by the parabola y2 = 4x, is
(1) 5 (2) 3 (3) 2 (4) 1

B-4. A variable chord PQ of the parabola, y2 = 4x is drawn parallel to the line y = x. If the parameters of the
points P & Q on the parabola be p & q respectively, then (p + q) equal to.
(1) 1 (2) 1/2 (3) 2 (4) 4

B-5. The length of the chord of the parabola, y2 = 12x passing through the vertex & making an angle of 60º
with the axis of x is:
(1) 8 (2) 4 (3) 16/3 (4) none of these

B-6. If one end of a focal chord of the parabola y2 = 4x is (1, 2), the other end is
(1) (1, – 2) (2) (2, 2) (3) (2, 1) (4) (–2, –1)

B-7. In the parabola y2 = 6x, the equation of the chord through vertex and negative end of latus rectum, is
(1) y = 2x (2) y + 2x = 0 (3) y + 3x = 0 (4) x + 2y = 0

Section-(C) : Tangent of Parabola

C-1. The value  such that line y = x +  is tangent to the parabola y2 = 8x


(1) 2 (2) 4 (3) 6 (4) 8

 1
C-2. If y = 2 x  3 is a tangent to the parabola y2 = 4a  x   , then ' a ' is equal to:
 3
14  14
(1) 1 (2)  1 (3) (4)
3 3

C-3. The equation of the tangent to the parabola y = (x  3)2 parallel to the chord joining the points (3, 0)
and (4, 1) is:
(1) 2 x  2 y + 6 = 0 (2) 2 y  2 x + 6 = 0 (3) 4 y  4 x + 11 = 0 (4) 4 x  4 y = 13
Conic Section

y
C-4. The value of a such that line = x – 1 is tangent to the parabola y2=6x, parallel to the line x+ y= 4.
a3 a
3
(1) – 4 (2) – 3 (3) – (4) – 2
2
2
C-5. The tangent drawn at any point P to the parabola y = 4ax meets the directrix at the point K, then angle
which KP subtends at its focus is
(1) 30º (2) 45º (3) 60º (4) 90º

C-6. Equation of a tangent to the parabola y2 = 12x which make an angle of 45° with line y = 3x + 77 is
(1) 2x – 4y + 3 = 0 (2) x + 2y + 12 = 0 (3) 4x + 2y + 3 = 0 (4) 2x + y – 12 = 0

C-7. The point where the tangent to the parabola y2 = 7x which is parallel to the straight line
4y – x + 3 = 0 touches the parabola is
(1) (28, 14) (2) (2, 3) (3) (–2, 1) (4) (0, 1)

C-8. The mirror image of the parabola y2 = 4x in the tangent to the parabola at the point (1, 2) is
(1) (x – 1)2 = 4(y – 2) (2) (x + 3)2 = 4(y + 2) (3) (x + 1)2 = 4(y – 1) (4) (x – 1)2 = 4 (y – 1)

C-9. The equation of tangent to the parabola y2 = 9x, which pass through the point (4, 10) is
(1) 4y = 9x + 4 (2) 4y = x – 36 (3) y = x + 36 (4) 4y = x + 32

Section-(D) : Normal, Pair of tangents, Director circle, Chord of conctact,chord with


given mid point
D-1. The equation of director circle of the parabola y2 = 10(x – 1)
(1) x2 + y2 –2x –4 y + 4 = 0 (2) 2x + 3 = 0
(3) 2x + 5 = 0 (4) x2 + y2 – 4x + 2y + 2 = 0

D-2. Equation of the normal to the parabola, y2 = 4ax at its point (am2, 2am) is:
(1) y =  mx + 2am + am3 (2) y = mx  2am  am3
(3) y = mx + 2am + am3 (4) none
D-3. At what point on the parabola y2 = 4x the normal makes equal angles with the axes?
(1) (4, 4) (2) (9, 6) (3) (4, – 1) (4) (1, 2)

D-4. The line 2x + y +  = 0 is a normal to the parabola y2 = – 8x, then  is


(1) 12 (2) – 12 (3) 24 (4) – 24

D-5. The distance between a tangent to the parabola y2 = 4 A x (A > 0) and the parallel normal with gradient
1 is :
(1) 4 A (2) 2 2 A (3) 2 A (4) 2A
2
D-6. If a line x + y = 1 cut the parabola y = 4ax in points A and B and normals drawn at A and B meet at C.
The normal to the parabola from C other, than above two meet the parabola in D, then point D is
(1) (a, a,) (2) (2a, 2a) (3) (3a, 3a) (4) (4a, 4a)

D-7. Number of distinct normals of a parabola passing through the focus of the parabola is
(1) 0 (2) 1 (3) 2 (4) 3

D-8. If y = 2x + c – 4 is a normal to the parabola y2 = 4x, then value of ‘c’ is


(1) 8 (2) – 8 (3) 12 (4) – 12
2
D-9. The equation of tangent drawn from the point (2,3) to the parabola y = 4x,are
(1) x–y+1 = 0, x+2y = 4 (2) x+y–1= 0, x+2y +4 = 0
(3) x–y+1= 0, x–2y +4 = 0 (4) x+y–1= 0, x+2y – 4 = 0
Conic Section

D-10. The angle between the tangents drawn from a point ( – a, 2a) to y2 = 4 ax is
   
(1) (2) (3) (4)
4 2 3 6

D-11. The angle between the tangents drawn from a point ( – a, 2a) to y2 = 4 ax is
   
(1) (2) (3) (4)
4 2 3 6

D-12. The line 4x  7y + 10 = 0 intersects the parabola, y2 = 4x at the points A & B. The co-ordinates of the
point of intersection of the tangents drawn at the points A & B are:
7 5  5 7 5 7  7 5
(1)  ,  (2)   ,   (3)  ,  (4)   ,  
2 2  2 2 2 2  2 2

D-13. The feet of the perpendicular drawn from focus upon any tangent to the parabola, y = x2  2x  3 lies on
(1) y + 4 = 0 (2) y = 0 (3) y = – 2 (4) y + 1 = 0

D-14.The locus of the middle points of the focal chords of the parabola, y2 = 4x is:
(1) y2 = x  1 (2) y2 = 2 (x  1) (3) y2 = 2 (1  x) (4) none of these

D-15. The equaton of chord of parabola y2=4x whose midpoint is (2,2)


(1) 2x – 2y + 5= 0 (2) y=x (3) 2y + 2x + 5= 0 (4) 2y – 2x + 5= 0

ELLIPSE
Section (A) : Standard

x2 y 2
A-1. Eccentricity of the conic   1 is
4 9
2 5 7 1
(1) (2) (3) (4)
3 3 3 2

x2 y2
A-2. The focii of the ellipse   1 are
25 9
(1) (±4, 0) (2) (±3,0) (3) (± 5,0) (4) (±2,0)

A-3. The equation of the ellipse whose focus is (1, –1), directrix is the line x – y – 3 = 0 and the eccentricity
1
is , is
2
(1) 7x2 + 2xy + 7y2 – 10x + 10y + 7 = 0 (2) 7x2 + 2xy + 7y2 + 7 = 0
2 2
(3) 7x + 2xy + 7y + 10x – 10y – 7 = 0 (4) none of these

A-4. The eccentricity of the ellipse 4x2 + 9y2 + 8x + 36y + 4 = 0 is


5 3 2 5
(1) (2) (3) (4)
6 5 3 3

A-5. If distance between the directrices be thrice the distance between the focii, then eccentricity of ellipse is
1 2 1 4
(1) (2) (3) (4)
2 3 3 5

A-6. The length of the latus rectum of the ellipse 9x2 + 4y2 = 1, is
3 8 4 8
(1) (2) (3) (4)
2 3 9 9
Conic Section

x y
A-7. The eccentricity of the ellipse which meets the straight line + = 1 on the axis of x and the straight
7 2
x y
line – = 1 on the axis of y and whose axes lie along the axes of coordinates is
3 5
6 4 6 2 6 2 6
(1) (2) (3) (4)
7 7 5 7

x2 y2
A-8. Let P be a variable point on the ellipse + = 1 with focii at S and S. If A be the area of triangle
25 16
PSS, then the maximum value of A is
(1) 24 sq. units (2) 12 sq. units (3) 36 sq. units (4) none of these

A-9. Equation of the ellipse whose foci are (2, 2) and (4, 2) and the major axis is of length 10 is
(x  3)2 (y  2)2 (x  3)2 (y  2)2
(1) + =1 (2) + =1
4 5 24 25
(x  3)2 (y  2)2 (x  3)2 (y  2)2
(3) + =1 (4) + =1
25 24 25 24

A-10. The length of the axes of the conic 9x2 + 4y2 – 6x + 4y + 1 = 0, are
1 2 2
(1) , 9 (2) 3, (3) 1, (4) 3, 2
2 5 3

x2 y2
A-11. The equation + +1 = 0 represents an ellipse, if -
2r r 5
(1) r > 2 (2) 2< r < 5 (3) r > 5 (4) r  {2, 5}

A-12. An ellipse has OB as semi-minor axis, F and F are its foci and FBF is a right angle then eccentricity
of the ellipse is
1 1 2 1
(1) (2) (3) (4)
2 2 3 3

A-13. The eccentricity of an ellipse in which distance between their focii is 10 and that of focus and
corresponding directrix is 15 is
1 1 1 1
(1) (2) (3) (4)
3 2 4 2

2 2
A-14. If P = (x, y), F1 = (3, 0), F2 = (3, 0) and 16x + 25y = 400, then PF1 + PF2 equals
(1) 8 (2) 6 (3) 10 (4) 12

1
A-15. If focus and corresponding directrix of an ellipse are (3, 4) and x + y – 1 = 0 and eccentricity is then
2
the co-ordinates of extremities of major axis are
(1) (2, 3), (4, 7) (2) (6, 7), (2, 3) (3) (1, 3), (2, 3) (4) (4, 7), (6, 7)
2 2
A-16. The equation, 3x + 4y  18x + 16y + 43 = C.
(1) cannot represent a real pair of straight lines for any value of C
(2) represents an ellipse, if C > 0
(3) no locus, if C < 0
(4) all of these
Conic Section

x2 y2
A-17. The distance of the point '' on the ellipse 2
  1 from a focus is
a b2
(1) a(e + cos ) (2) a(e – cos ) (3) a(1 + e cos ) (4) a(1 + 2e cos )

A-18. The curve represented by x = 3 (cos t + sin t), y = 4 (cos t – sin t), is
(1) ellipse (2) parabola (3) hyperbola (4) circle

Section (B) : Position of Point, Chord/Tangent of Ellipse

x2 y 2
B-1. The position of point (4, 3) with respect to the ellipse   1 is
8 9
(1) Out side the ellipse (2) In side the ellipse
(3) On the ellipse (4) On the major axis of ellipse
2 2
B-2. The position of the point (1, 3) with respect to the ellipse 4x + 9y – 16x – 54y + 61 = 0 is
(1) outside the ellispe (2) on the ellipse (3) on the major axis (4) on the minor axis

x2 y2
B-3. If the line y = 2x + c be a tangent to the ellipse + = 1, then c is equal to
8 4
(1) ± 4 (2) ± 6 (3) ± 1 (4) ± 8

B-4. If the line 3x + 4y =  7 touches the ellipse 3x2 + 4y2 = 1 then, the point of contact is
 1 1   1 1   1 1   1  1 
(1)  ,  (2)  ,  (3)  ,  (4)  , 
 7 7  3 3  7 7  7 7

x 2 y2
B-5 The equation of tangent to the ellipse + =1 which passes through a point (15, – 4) is
50 32
(1) 4x + 5y = 40 (2) 4x + 35y = 200 (3) 4x – 5y = 40 (4) none of these

x2 y2
B-6. The tangents at the point P on the ellipse  = 1 and its corresponding point Q on the auxiliary
a2 b2
circle meet on the line :
(1) x = a/e (2) x = 0 (3) y = 0 (4) x = –a/e

x y x2 y2
B-7. If + = 2 touches the ellipse 2 + 2 =1 at a point P, then eccentric angle of P is
a b a b
(1) 0 (2) 45° (3) 60° (4) 90°

B-8. The equation of the tangents drawn at the ends of the major axis of the ellipse 9x2 + 5y2 – 30y = 0, are
(1) y = ± 3 (2) x = ± 5 (3) y = 0, y = 6 (4) none of these

x2 y2
B-9. The minimum area of triangle formed by the tangent to the ellipse + =1 and coordinate axes is
a2 b2
a2  b2
(1) ab sq. units (2) sq. unit
2
(a  b)2 a2  ab  b2
(3) sq. units (4) sq. units
2 3
Conic Section

Section (C) : Normal, Pair of Tangents, Director circle, Chord of conctact,


chord with given mid point of ellipse

8 y2
C-1. The value of , for which the line 2x – y = – 3 is a normal to the conic x2 + = 1 is
3 4
3 1 3 3
(1) ± (2) ± (3) – (4) ±
2 2 4 8
x 2 y2
C-2. The eccentric angle of the point where the line, 5x – 3y = 8 2 is a normal to the ellipse + =1 is
25 9
   
(1) (2) (3) (4)
4 2 3 6

x2 y2
C-3. The equation of the normal to the ellipse + = 1 at the positive end of latus rectum is
a2 b2
(1) x + ey + e2a = 0 (2) x – ey – e3a = 0 (3) x – ey – e2a = 0 (4) none of these

x2 y 2
C-4. If the normal at the point P() to the ellipse  = 1 intersects it again at the point Q(2), then
14 5
cosis equal to
2 2 3 3
(1) (2) – (3) (4) –
3 3 2 2

x2 y2
C-5. If the normal at an end of a latus-rectum of an ellipse +
= 1 passes through one extremity of the
a2 b2
minor axis, then the eccentricity of the ellipse is given by the relation
3
(1) e4 + 2e2 – 4 = 0 (2) e4 + e2 – 1 = 0 (3) e4 + e2 – =0 (4) e4 – e2 – 1 = 0
2
x2 y 2
C-6. The pair of tangents drawn from the point (4,3) to the ellipse   1 is
16 9
(1) 3x + 4y – xy – 12 = 0 (2) x2 + y2 –3x+ 4y – 12 = 0
(3) 4x + 3y + xy + 12 = 0 (4) x2 + y2 + 2xy +2x+2y+1= 0

x 2 y2
C-7. Equation of director circle of the ellipse  1
25 16
2 2 2 2 2 2 2 2
(1) x + y = 25 (2) x + y = 16 (3) x + y = 41 (4) x + y = 9
x2 y2
C-8. A triangle ABC right angled at 'A' moves so that its sides touch the curve 2
+ =1 all the time. The
a b2
locus of the point 'A' is
(1) x2 + y2 = 2a2 2 2
(2) x + y = 2b
2 2 2
(3) x + y = a + b
2 2
(4) none of these
x 2 y2
C-9. If 3x + 4y = 12 intersect the ellipse + = 1 at P and Q, then the point of intersection of tangents at
25 16
P and Q is
 25 16   25 16 
(1) (0, 1) (2) (1, –2) (3)  ,  (4)  – , 
 4 3   4 3 
2 2
C-10. The angle between the pair of tangents drawn to the ellipse 3x + 2y = 5 from the point (1, 2) is
 12   12 
(1) tan–1   (2) tan–1 (6 5 ) (3) tan–1   (4) tan–1 (12 5 )
 5   5
Conic Section

C-11. If F1 & F2 are the feet of the perpendiculars from the focii S1 & S2 of an ellipse
x2 y2
 = 1 on the tangent at any point P on the ellipse, then (S1F1). (S2F2) is equal to :
5 3
(1) 2 (2) 3 (3) 4 (4) 5

(x  1)2 (y  2)2
C-12. A ray emanating from (6, 2) is incident on ellipse + = 1 at (4, 6). The equation of
45 20
st
reflected ray (after 1 reflection) is
(1) x – 2y + 8 = 0 (2) x + 2y + 8 = 0 (3) x + 2y – 8 = 0 (4) x – 2y – 8 = 0

1 
C-13. The equation of chord of the ellipse 2x2+y2= 2 whose midpoint is  ,1
2 
3 3 5 5
(1) x+y = (2) 2x+y = (3) x+2y = (4) x+y =
2 2 2 2

HYPERBOLA
Section- (A) : Elementary Concepts of Hyperbola/Conjugate/ Rectangular Hyperbola(xy= c2)
A-1. The eccentricity of the conic represented by x2 – y2 – 4x + 4y + 16 = 0 is
(1) 1 (2) 2 (3) 2 (4) 1/2

A-2. Which of the following pair, may represent the eccentricities of two conjugate hyperbolas, for all
 (0, /2) ?
(1) sin , cos  (2) tan , cot  (3) sec , cosec  (4) 1 + sin , 1 + cos 

x2 y 2
A-3. The length of latus rectum of the hyperbola – = 1, is
9 16
16 32
(1) (2) (3) 8 (4) 6
3 3

A-4. The eccentricity of the hyperbola whose latus rectum is 8 and conjugate axis is equal to half the
distance between the focii, is :
4 4 2
(1) (2) (3) (4) none of these
3 3 3

A-5. The equation of the hyperbola whose conjugate axis is 5 and the distance between the focii is 13, is
(1) 25x2 – 144 y2 = 900 (2) 144 x2 – 25 y2 = 900
2 2
(3) 144 x + 25 y = 990 (4) 25x2 + 144 y2 = 900

A-6. The length of the transverse axis of a hyperbola is 7 and it passes through the point (5, –2). The
equation of the hyperbola is
4 2 196 2 4 2 196 2 4 2 51 2
(1) x– y =1 (2) x – y =1 (3) x – y =1 (4) None of these
49 51 49 51 49 196
A-7. The equation of the hyperbola whose directrix is x + 2y = 1, focus (2,1) and eccentricity 2 will be
2 2 2 2
(1) x – 16xy – 11 y – 12x + 6y + 21 = 0 (2) 3x + 16xy + 15 y – 4x – 14y – 1 = 0
2 2
(3) 3x + 16xy + 11 y – 12x – 6y + 21 = 0 (4) None of these

A-8. The vertices of a hyperbola are at (0, 0) and (10, 0) and one of its foci is at (18, 0). The equation of the
hyperbola is
x2 y2 (x – 5)2 y2 x 2 (y – 5)2 (y – 5)2 y2
(1) – =1 (2) – = 1 (3) – =1 (4) – =1
25 144 25 144 25 144 25 144
Conic Section

A-9. The equation of the directrices of the conic x2 + 2x – y2 + 5 = 0 are


(1) x = ± 1 (2) y = ± 2 (3) y = ± 2 (4) x = ± 3

2 2 2 2
A-10. If e and eare the eccentricities of the ellipse 5x + 9y = 45 and the hyperbola 5x – 4y = 45
respectively then ee =
(1) – 1 (2) 1 (3) – 4 (4) 9

x2 y2
A-11. The equation of auxilary circle of hyperbola – 1
9 16
2 2 2 2
(1) x + y = 9 (2) x + y = 16 (3) x2 + y2 = 41 (4) x2 + y2 = 7

A-12. An ellipse and a hyperbola have the same centre origin, the same foci and the minor-axis of the one is
1 1
the same as the conjugate axis of the other. If e1, e2 be their eccentricities respectively, then 2  2 =
e1 e2
(1) 1 (2) 2 (3) 4 (4) none of these

x2 y2 y2 x2
A-13. If e and e are the eccentricities of the hyperbola 2
– 2
= 1 and 2
– = 1, then the point
a b b a2
1 1 
 e , e  lies on the circle :
 
(1) x2 + y2 = 1 (2) x2 + y2 = 2 (3) x2 + y2 = 3 (4) x2 + y2 = 4

A-14. If P ( 2 sec , 2 tan ) is a point on the hyperbola whose distance from the origin is 6 where P is
in the first quadrant then  =
  
(1) (2) (3) (4) None of these
4 3 6

x2 (y – 2)2
A-15. Foci of the hyperbola – = 1 are
16 9
(1) (5, 2), (–5, 2) (2) (5, 2), (5.–2) (3) (5, 2), (–5, –2) (4) None of these

x 2 y2 x2 y2
A-16. The ellipse   1 and the hyperbola   1 have in common
25 16 25 16
(1) centre only (2) centre, foci and directries
(3) Centre, foci and vertices (4) centre and vertices only

 c 2
A-17.If the normal at  ct,  on the curve xy = c meets the curve again at t, then
 t 
1 1 1 1
(1) t = – (2) t = (3) t = (4) t2 = –
t3 t t2 t2

A-18.If (, 4) is the orthocentre of the triangle whose vertices lie on the rectangular hyperbola xy = 16, then
 is equal to
(1) 3 (2) 4 (3) 12 (4) 8

A-19.If (5, 12) and (24, 7) are the focii of a conic passing through the origin then the eccentricity of conic is
(1) 386 /12 (2) 386 /13
(3) 386 /25 (4) 386 /38 or ;k 386 /12
Conic Section

Section-(B) : Position of Point/Line, Tangent, Chord of hyperbola

B-1. The line x + y = a touches the hyperbola x2 – 2y2 = 18, if a is equal to ± b, then value of | b | is
(1) 3 (2) 4 (3) 12 (4) 8

B-2. Equation of a tangent passing through (2, 8) to the hyperbola 5 x2  y2 = 5 is :


(1) 3 x  y + 2 = 0 (2) 3 x + y – 14 = 0 (3) 23 x  3 y  22 = 0 (4) 3 x  23 y + 178 = 0

x2 y2
B-3. Tangent at any point on the hyperbola –
= 1 cut the axes at A and B respectively. If the
a2 b2
rectangle OAPB (where O is origin) is completed then locus of point P is given by
a2 b2 a2 b2 a2 b2
(1) – 2
=1 (2) + 2
=1 (3) 2
– =1 (4) none of these
x2 y x2 y y x2

B-4. The number of possible tangents which can be drawn to the curve 4x2  9y2 = 36, which are
perpendicular to the straight line 5x + 2y 10 = 0 is :
(1) zero (2) 1 (3) 2 (4) 4

B-5. The equation of the tangent lines to the hyperbola x2  2y2 = 18 which are perpendicular to the line y =
x are :
(1) y = – x + 7 (2) y = x + 3 (3) y = – x – 4 (4) y = – x ± 3

B-6. Number of non-negative integral values of b for which tangent parallel to line y = x + 1 can be drawn to
hyperbola
x2 y2
 = 1 is
5 b2
(1) 16 (2) 2 (3) 3 (4) 4

B-7. An equation of a tangent to the hyperbola, 16x2 – 25y2 – 96x + 100y – 356 = 0 which makes an angle

with the transverse axis is y = x +  , ( > 0), then 2 is
4
(1) 16 (2) 4 (3) 3 (4) 9

x2 y2 
B-8. If (a sec , b tan ) and (a sec, b tan ) are the ends of a focal chord of 2
– 2
= 1, then tan
a b 2

tan equals to
2
e 1 e 1 1 e
(1) (2) (3) (4) none of these
e 1 e 1 1 e

2 2
B-9. The locus of the middle points of chords of hyperbola 3x – 2y + 4x – 6y = 0 parallel to y = 2x is
(1) 3x – 4y = 4 (2) 3y – 4x + 4 = 0 (3) 4x – 4y = 3 (4) 3x – 4y = 2

x2 y2
B-10. The chords passing through L(2, 1) intersects the hyperbola – =1 at P and Q. If the tangents at
16 9
P and Q intersects at R then Locus of R is
(1) x – y = 1 (2) 9x – 8y = 72 (3) x + y = 3 (4) None of these
Conic Section

Section- (C): Normal/Director Circle / Chord of Contact/Chord with given midpoint/ pair
of tangent of hyperbola

C-1. The tangents from (1, 2 2 ) to the hyperbola 16x2 – 25y2 = 400 include between them an angle equal
to:
   
(1) (2) (3) (4)
6 4 3 2

C-2. The number of points from where a pair of perpendicular tangents can be drawn to the hyperbola,
x2 sec2   y2 cosec2  = 1,  (0, /4), is :
(1) 0 (2) 1 (3) 2 (4) infinite

C-3. The product of the lengths of the perpendiculars from the two focii on any tangent to the hyperbola
x2 y2
– = 1 is k , then k is
25 3
(1) 16 (2) 4 (3) 3 (4) 9

C-4. The equation to the locus of the feet of the perpendicular from the focus of the hyperbola
2 2
4x  9y = 36 upon any of its tangent has the equation
(1) x2 + y2 = 9
2 2
(2) x + y = 4 (3) x2 + y2 = 1 (4) x2 + y2 = 16

C-5. From Point p(2,3) two tangents PA and PB are drawn to the hyperbola x2–y2 – 4x+4y + 16 = 0. The
equation of line AB is
(1) y = 3 (2) y = 2 (3) x = 1 (4) x = 3

C-6. The equation of chord of the hyperbola x2–2y2 = 2 whose midpoint is (3,1)
(1) 3x –2y+7 = 0 (2) 2x –3y+7 = 0 (3) 2x –3y=7 (4) 3x –2y=7

Section-(D): Problems involving more than one conic

x2 y 2
D-1. x  2y + 4 = 0 is a common tangent to y2 = 4x &  = 1. Then the value of ‘b’ and the other
4 b2
common tangent are respectively
(1) b = 3 ; x + 2y + 4 = 0 (2) b = 3; x + 2y + 4 = 0
(3) b = 3 ; x + 2y  4 = 0 (4) b = 3 ; x  2y  4 = 0

D-2. The equation of common tangent of x2 + y2 = 2 and y2 = 8x is


(1) x – y + 2 = 0 (2) x + y + 1 = 0 (3) x – y + 1 = 0 (4) x + y – 2 = 0

x2 y 2
D-3. The line y = x intersects the hyperbola – = 1 at the points P and Q. The eccentricity of ellipse
9 25
5
with PQ as major axis and miror axis of length is
2
5 5 5 2 2
(1) (2) (3) (4)
3 3 9 3

D-4. The equation of common normal to the circle x2 + y2 – 12x + 16 = 0 and parabola y2 = 4x is
(1) y = 0 (2) 2x – y = 12 (3) 2x + y = 12 (4) All of the above
Conic Section

D-5. Equation of common tangent to circle x2 + y2 = 5 and ellipse x2 + 9y2 = 9 is


(1) x – y + 10 = 0 (2) x + y = 10 (3) x – y = 10 (4) All of the above
2 2 2 2
D-6. Equation of common tangent to ellipse 5x + 2y = 10 , and hyperbola 11x – 3y = 33 is
(1) y = ± 3x ± 21 (2) y = ± x ± 3 (3) y = ± 4x ± 37 (4) 3x ± y = 12

x2 y2 2
D-7. If latus rectum of ellipse   1 is double ordiante of parabola y = 4ax value of a is
25 16
64 25 64 64
(1) (2) (3) (4)
25 64 50 75

D-8. The equation of common tangent to the parabola y2 = 8x and hyperbola 3x2 – y2= 3 is
(1) 2x ± y + 1 = 0 (2) 2x ± y – 1 = 0 (3) x ± 2y + 1 = 0 (4) x ± 2y – 1 = 0

 Marked Questions may have for Revision Questions.


* Marked Questions may have more than one correct option.

PART - I : OBJECTIVE QUESTIONS


1. The focus of the parabola x2 + 2y – 3x + 5 = 0 is
 3 15   3 15   3 15   3 15 
(1)  ,  (2)  , (3)  – , (4)  – , –
2 8  2 8 

 2 8 

 2 8 

2. Length of the focal chord of the parabola y2 = 4ax at a distance p from the vertex is:
2a2 a3 4a3 p2
(1) (2) (3) (4)
p p2 p2 a
3. If the segment intercepted by the parabola y2 = 4ax with the line x + my + n = 0 subtends a right angle
at the vertex, then
(1) 4a+ n = 0 (2) 4a+ 4am + n = 0 (3) 4am + n = 0 (4) None of these

4. The tangents at the extremities of a focal chord of a parabola


(1) intersect at the vertex (2) are parallel
(3) intersect on the directrix (4) none of these
2
5. The locus of the mid point of the focal radii of a variable point moving on the parabola, y = 4ax is a
parabola whose
(1) Latus rectum is half the latus rectum of the original parabola
(2) Vertex is (a/2, 0)
(3) Directrix is y-axis
(4) All of these
2 2 2
6. Let y = 4ax be a parabola and x + y + 2 bx = 0 be a circle. If parabola and circle touch each other
externally then:
(1) a > 0, b > 0 (2) a > 0, b < 0 (3) a < 0, b > 0 (4) ab > 0

7. Length of common chord of the parabolas x2 = 4y and y2 = 4x is


(1) 4 (2) 4 3 (3) 4 2 (4) 2

8. The circle x2 + y2 = 5 meets the parabola y2 = 4x at P & Q. Then the length PQ is equal to
(1) 1 (2) 2 (3) 3 (4) 4
Conic Section

9. The circles on focal radii of a parabola as diameter touch:


(1) the tangent at the vertex (2) the axis
(3) the directrix (4) none of these
2
10. A circle described on any focal chord of the parabola, y = 4ax as its diameter will touch
(1) the axis of the parabola
(2) the directrix of the parabola
(3) the tangent drawn at the vertex of the parabola
(4) latus rectum

x2 y2
11. The distance between the directrices of the ellipse + = 1 is
36 20
(1) 18 (2) 9 (3) 27 (4) 5

5
12. If the eccentricity of an ellipse be and the distance between its focii be 10, then its latus rectum is
8
39 39 39 39
(1) (2) (3) (4)
4 8 2 2
1
13. If the distance between a focus and corresponding directix of an ellipse be 8 and the eccentricity be ,
2
then length of the semi minor axis is
8 4
(1) (2) (3) 8 3 (4) 4 3
3 3
x2 y2
14. Sum of distances of a point on the ellipse + =1 from the focii is
9 16
(1) 2 (2) 4 (3) 8 (4) 16

15. Equation of circle centered at extremity of positive minor axis and passing through the focus (on
x2 y2
positive x-axis) of the ellipse + = 1 is
16 9
(1) x2 + y2 – 6y – 7 = 0 (2) x2 + y2 – 6y = 0
(3) x2 + y2 – 6y – 16 = 0 (4) x2 + y2 – 3x – 3y = 0

x2 y2
16. If a point P(x1, y1) lies on ellipse + = 1, then focal distance of P is
9 25
4 4 4 4
(1) 4 + y (2) 3 + y (3) 4 – y (4) 5 – y
5 1 5 1 5 1 5 1

17. A latus rectum of an ellipse is a line


(1) passing through a focus (2) parallel to the major axis
(3) perpendicular to the minor axis (4) all of these

x2 y2
18. + =1 will represents the ellipse, if r lies in the interval :
r 2 – r – 6 r 2 – 6r  5
(1) (– , ) (2) (3, ) (3) (5, ) (4) (1, )

19. The equations of the common tangent to the ellipse, x2 + 4y2 = 8 & the parabola y2 = 4x is
(1) 2y – x = 8 (2) 2y + x = 4 (3) 2y + x + 4 = 0 (4) 2y + x = 0
Conic Section

20. The total number of real common tangents that can be drawn to the ellipse 3x 2 + 5y2 = 32 and
2 2
25x + 9y = 450 passing through (3, 5) is
(1) 1 (2) 2 (3) 0 (4) 3

x2 y2
21. If the mid point of a chord of the ellipse + = 1 is (0, 3), then length of the chord is
16 25
32 32 8 16
(1) (2) (3) (4)
3 5 5 5

22. A ray emanating from the point ( 4, 0) is incident on the ellipse 9x² + 25y² = 225 at the point P with
abscissa 3. The equation of the reflected ray after first reflection is
(1) 12 x + 5 y = 48 (2) 5x  12 y = 48 (3) 12 x  5 y = 24 (4) all of these

23. Identify the correct statement(s) -


(1) the equation of the director circle of the ellipse, 5x 2 + 9y2 = 45 is x2 + y2 = 14.
x2 y2
(2) the sum of the focal distances of the point (0, 6) on the ellipse+ = 1 is 10.
25 36
(3) the point of intersection of any tangent to a parabola & the perpendicular to it from the focus
lies on directrix.
(4) all of these

(x  y – 1)2 (x – y  2)2
24. Find the centre of the ellipse + =1
2 3
 1 3  3 1  1 3 1 3
(1)  – ,  (2)   (3)  – , –  (4)  , – 
 2 2  2 2  2 2  2 2

25. The centre of ellipse 5x2+5y2– 2xy +8x+8y+2=0 is


(1) (–1, 2) (2) (–2, 1) (3) (–2,–2) (4) (–1,–1)

x2 y2
26. The foci of a hyperbola coincide with the foci of the ellipse + = 1. The equation of the hyperbola
25 9
if its eccentricity is 2 is
2 2 2 2
(1) 3x – y – 4 = 0 (2) 3x – y – 16 = 0
(3) 3x2 – y2 – 12 = 0 (4) – 3x2 + y2 + 20 = 0

x2 y2
27. The hyperbola   1 passes through the point of intersection of the lines,
a2 b2
7x + 13y  87 = 0 & 5x  8y + 7 = 0 & the latus rectum is 32 2 /5. The value of ab is
(1) 10 2 (2) 5 2 (3) 10 3 (4) 200

28. The one which does not represent a hyperbola is


(1) xy = 1 (2) x2 – y2 = 5 (3) (x –1) (y – 3) = 3 (4) x2 – y2 = 0

29. The latus rectum of the hyperbola


9x2 – 16 y2 – 18 x – 32y – 151 = 0 is
9 3 9
(1) (2) 9 (3) (4)
4 2 2
2 2
30. The co-ordinates of a focus of the hyperbola 9x – 16y + 18 x + 32y – 151 = 0 are
(1) (–1, 1), (4, 1) (2) (6, 1), (–6, 1) (3) (4, 1), (6, 1) (4) (– 6, 1), (4, 1)
Conic Section

31. The equation of the hyperbola with vertices (3, 0) and (–3, 0) and semi-latusrectum 4, is given by is
4x2 – 3y2 = 4k, then k =
(1) 16 (2) 4 (3) 3 (4) 9
2 2 2
32. If the eccentricity of the hyperbola x  y sec  = 5 is 3 times the eccentricity of the ellipse
x2 sec2  + y2 = 25, then a value of  is
(1) /2 (2) /3 (3) /4 (4) /6

x2 y2
33. The length of the latus rectum of the hyperbola 2
– = – 1 is
a b2
2a2 a2 a2 a
(1) (2) (3) (4)
b b 2b 2b2

x2 y2 x2 y2
34. If hyperbola – = 1 passes through the focus of ellipse + = 1 then eccentricity of
b2 a2 a2 b2
hyperbola is.
(1) 3 (2) 2 (3) 5 (4) 2

(x  1)2 (y  2)2
35. The co-ordinates of the focii of the hyperbola  = 1 are
9 16
(1) (6, 2) and (–6, 2) (2) (– 6, 2) and (4, 2) (3) ( 6, 2) and (– 4, 2) (4) (6, – 2) and (–4, 2)

36. The equation x2 + 4 xy + y2 + 2x + 4y + 2 = 0


(1) An ellipse (2) A pair of straight lines (3) A hyperbola (4) None of these
37. A rectangular hyperbola circumscribe a triangle ABC, then it will always pass through its
(1) orthocenter (2) circumcentre (3) centroid (4) incentre

PART - II : MISCELLANEOUS QUESTIONS


Section (A) : ASSERTION/REASONING
DIRECTIONS :
Each question has 4 choices (1), (2), (3) and (4) out of which ONLY ONE is correct.
(1) Both the statements are true.
(2) Statement- is true, but Statement- is false.
(3) Statement- is false, but Statement- is true.
(4) Both the statements are false.

A-1. STATEMENT-1 : Normal chord drawn at the point (8, 8) of the parabola y2 = 8x subtends a right angle
at the vertex of the parabola.
STATEMENT-2 : Every chord of the parabola y2 = 4ax passing through the point (4a, 0) subtends a
right angle at the vertex of the parabola.
A-2. Statement -1 : The circle on any focal distance as diameter touches the director circle
x2 y2
Statement - 2 : If P be any point on the ellipse + = 1 (a < b) with S & S as its focii, then
a2 b2
(SP) + (SP) = 2b
(SP) + (SP) = 2b
x2 y2
A-3. STATEMENT-1 : Total number of tangents of the hyperbola – = 1 that are perpendicular
9 4
to the line 5x + 2y – 3 = 0 is 2.
x2 y2
STATEMENT-2 : The equation of tangent to the hyperbola – 1 in terms of its slope m is
a2 b2
2 2 2
y = mx ± a2m2 – b2 , if a m – b > 0.
Conic Section

Section (B) : MATCH THE COLUMN


B-1. Column - I Column - II

(P) Number of positive integral values of b for which tangent (1) 16


parallel to line y = x + 1 can be drawn to hyperbola
x2 y2
 = 1 is
5 b2
(Q) The equation of the hyperbola with vertices (3, 0) and (2) 2
(–3, 0) and semi-latusrectum 4, is given by is
4x2 – 3y2 = 4k, then k =
(R) The product of the lengths of the perpendiculars (3) 4
from the two focii on any tangent to the hyperbola
x2 y2
– = 1 is k , then k is
25 3

(S) An equation of a tangent to the hyperbola, (4) 9


2 2
16x – 25y – 96x + 100y – 356 = 0 which makes an

angle with the transverse axis is y = x +  ,  then 2 is
4
Codes :
P Q R S
(1) 1 4 3 4
(2) 2 4 3 3
(3) 2 4 4 3
(4) 4 2 3 1

Section (C) : ONE OR MORE THAN ONE OPTIONS CORRECT

C-1. If one end of a focal chord of the parabola y2 = 4x is (1, 2), the other end lies on
(1) x2 y + 2 = 0 (2) xy + 2 = 0 (3) xy – 2 = 0 (4) x2 + xy – y – 1 = 0

2 2
C-2. The equation, 3x + 4y  18x + 16y + 43 = C.
(1) cannot represent a real pair of straight lines for any value of C
(2) represents an ellipse, if C > 0
(3) no locus, if C < 0
(4) a point, if C = 0

C-3. If the distance between the focii of an ellipse is equal to the length of its latus rectum, then eccentricity
of the ellipse is :
5 1 5 1 5 2 2
(1) (2) (3) (4)
2 2 2 5 1

C-4. The co-ordinates of a focus of the hyperbola 9x2 – 16y2 + 18 x + 32y – 151 = 0 are
(1) (–1, 1) (2) (6, 1) (3) (4, 1) (4) (– 6, 1)
Conic Section

 Marked Questions may have for Revision Questions.


* Marked Questions may have more than one correct option.
PART - I : JEE (MAIN) / AIEEE PROBLEMS (PREVIOUS YEARS)
1. Let P be the point (1, 0) and Q be a point on the curve y2 = 8x. The locus of midpoint of PQ is
[AIEEE 2005 (3, –1), 225]
(1) x2 – 4y + 2 = 0 (2) x2 + 4y + 2 = 0 (3) y2 + 2x + 2 = 0 (4) y2 – 4x + 2 = 0

2. An ellipse has OB as semi minor axis, F and F´ as foci and the angle FBF´ is a right angle. Then the
eccentricity of the ellipse is : [AIEEE 2005 (3, –1), 225]
1 1 1 1
(1) (2) (3) (4)
3 4 2 2
1
3. A focus of an ellipse at the origin. The directrix is the line x = 4 and eccentricity is , then the length of
2
the semi major axis is [AIEEE 2005 (3, –1), 225]
8 2 4 5
(1) (2) (3) (4)
3 3 3 3

4. The locus of a point P(,) moving under the condition that the line y = x +  is a tangent to the
x2 y2
hyperbola   1 is : [AIEEE 2005 (3, –1), 225]
a2 b2
(1) a hyperbola (2) a parabola (3) a circle (4) an ellipse

a3 x 2 a2 x
5. The locus of the vertices of the family of parabolas y = + – 2a is :
3 2
[AIEEE-2006 (3, –1), 165]
3 35 64 105
(1) xy = (2) xy = (3) xy = (4) xy =
4 16 105 64

6. In an ellipse, the distances between its foci is 6 and minor axis is 8. Then its eccentricity is :
[AIEEE 2006 (3, –1), 120]
1 4 1 3
(1) (2) (3) (4)
2 5 5 5

7. The equation of a tangent to the parabola y2 = 8x is y = x + 2. The point on this line from which the
other tangent to the parabola is perpendicular to the given tangent is [AIEEE 2007 (3, –1), 120]
(1) (–1, 1) (2) (0, 2) (3) (2, 4) (4) (–2, 0)

x2 y2
8. For the hyperbola – = 1, which of the following remains constant when  varies ?
cos2  sin2 
[AIEEE 2007 (3, –1), 120]
(1) Eccentricity (2) Directrix
(3) Abscissae of vertices (4) Abscissae of foci
2 2
9. The ellipse x + 4y = 4 is inscribed in a rectangle alingent with the coordinate axes, which in turn is
inscribed in another ellipse that passes through the point (4, 0). Then the equation of the ellipse is :
[AIEEE 2009 (4, –1), 144]
(1) x2 + 12y2 = 16 (2) 4x2 + 48y2 = 48 (3) 4x2 + 64y2 = 48 (4) x2 + 16y2 = 16
Conic Section

10*. Equation of the ellipse whose axes are the axes of coordinates and which passes through the point
2
(–3, 1) and has eccentricity is : [AIEEE 2011, I, (4, –1), 120]
5
(1) 3x2 + 5y2 – 32 = 0 (2) 5x2 + 3y2 – 48 = 0
2 2
(3) 3x + 5y – 15 = 0 (4) 5x2 + 3y2 – 32 = 0

11. The equation of the hyperbola whose foci are (–2, 0) and (2, 0) and eccentricity is 2 is given by :
[AIEEE 2011, II, (4, –1), 120]
(1) x2 – 3y2 = 3 (2) 3x2 – y2 = 3 (3) – x2 + 3y2 = 3 (4) – 3x2 + y2 = 3

12. An ellipse is drawn by taking a diameter of the circle (x – 1)2 + y2 = 1 as its semi-minor axis and a
diameter of the circle x2 + (y – 2)2 = 4 is semi-major axis. If the centre of the ellipse is at the origin and
its axes are the coordinate axes, then the equation of the ellipse is : [AIEEE-2012, (4, –1)/120]
2 2 2 2 2 2
(1) 4x + y = 4 (2) x + 4y = 8 (3) 4x + y = 8 (4) x2 + 4y2 = 16

13. Statement-1 : An equation of a common tangent to the parabola y2 = x and the ellipse 2x2 + y2 = 4 is
y = 2x + 2 3 [AIEEE - 2013, (4, – 1) 120 ]
4 3
Statement-2 : If the line y = mx + , (m  0) is a common tangent to the parabola y2 = 16 3 x
m
and the ellipse 2x2 + y2 = 4, then m satisfies m4 + 2m2 = 24.
(1) Statement-1 is false, Statement-2 is true.
(2) Statement-1 is true, statement-2 is true; statement-2 is a correct explanation for Statement-1.
(3) Statement-1 is true, statement-2 is true; statement-2 is not a correct explanation for Statement-1.
(4) Statement-1 is true, statement-2 is false.

x2 y 2
14. The equation of the circle passing through the foci of the ellipse  = 1, and having centre at
16 9
(0, 3) is [AIEEE - 2013, (4, – 1) ]
(1) x2 + y2 – 6y – 7 = 0
2 2
(2) x + y – 6y + 7 = 0
(3) x2 + y2 – 6y – 5 = 0 (4) x2 + y2 – 6y + 5 = 0

15. The locus of the foot of perpendicular drawn from the centre of the ellipse x2 + 3y2 = 6 on any tangent to
it is:
(1) (x2 + y2)2 = 6x2 + 2y2
2 2 2 2 2
(2) (x + y ) = 6x – 2y
2 2 2 2 2 2 2 2 2 2
(3) (x – y ) = 6x + 2y (4) (x – y ) = 6x – 2y
2 2
16. The slope of the line touching both the parabolas y = 4x and x = – 32y is :
[JEE(Main) 2014, (4, – 1), 120]
1 2 1 3
(1) (2) (3) (4)
8 3 2 2

17. The area (in sq.units) of the quadrilateral formed by the tangents at the end points of the latera recta to
x2 y 2
the ellipse   1, is [JEE(Main) 2015, (4, – 1), 120]
9 5
27 27
(1) (2) 18 (3) (4) 27
4 2
18. Let O be the vertex and Q be any point on the parabola, x 2 = 8y. If the point P divides the line segment
OQ internally in the ratio 1 : 3, then the locus of P is [JEE(Main) 2015, (4, –1), 120]
2 2 2
(1) x = y (2) y = x (3) y = 2x (4) x2 = 2y
Conic Section

19. Let P be the point on the parabola, y2 = 8x which is at a minimum distance from the centre C of the
2 2
circle, x + (y + 6) = 1. Then the equation of the circle, passing through C and having its centre at P is :
[JEE(Main) 2016, (4, – 1), 120]%
2 2
(1) x + y – x + 4y –12 = 0 (2) x2 + y2 – + 2y –24 = 0
(3) x2 + y2 –4 x + 9y + 18 = 0 (4) x2 + y2 – 4x + 8y +12 = 0

20. The eccentricity of the hyperbola whose length of the latus rectum is equal to 8 and the length of its
conjugate axis is equal to half of the distance between its foci, is :
[JEE(Main) 2016, (4, – 1), 120]
4 2 4
(1) (2) (3) 3 (4)
3 3 3
1
21. The eccentricity of an ellipse whose centre is at the origin is . If one of its directrices is x = – 4, then
2
 3
the equation of the normal to it at  1,  is [JEE(Main) 2017, (4, – 1), 120]
 2
(1) 2y – x = 2 (2) 4x – 2y = 1 (3) 4x + 2y = 7 (4) x + 2y = 4

22. A hyperbola passes through the point P( 2 , 3 ) and has foci at (±2, 0). Then the tangent to this
hyperbola at P also passes through the point : [JEE(Main) 2017, (4, – 1), 120]
(1) (3 2 , 2 3 ) (2) (2 2 , 3 3 ) (3) ( 3 , 2 ) (4) (– 2 , – 3 )

23. If the tangent at (1, 7) to the curve x 2 = y – 6 touches the circle x2 + y2 + 16x + 12y + c = 0 then the
value of c is : [JEE(Main) 2018, (4, – 1), 120]
(1) 85 (2) 95 (3) 195 (4) 185

24. Tangents are drawn to the hyperbola 4x2 – y2 = 36 at the points P and Q. If these tangents intersect at
the point T(0, 3) then the area (in sq. units) of PTQ is : [JEE(Main) 2018, (4, – 1), 120]
(1) 60 3 (2) 36 5 (3) 45 5 (4) 54 3

25. Tangent and normal are drawn at P(16,16) on the parabola y2 = 16x, which intersect the axis of the
parabola at A and B, respectively. If C is the centre of the circle through the points P, A and B and
CPB =  , then a value of tan  is : [JEE(Main) 2018, (4, – 1), 120]
4 1
(1) 3 (2) (3) (4) 2
3 2
PART - II : JEE (ADVANCED) / IIT-JEE PROBLEMS (PREVIOUS YEARS)
1. The locus of the mid point of the line segment joining the focus to a moving point on the parabola
2
y = 4ax is another parabola with directrix [IIT-JEE 2002, Scr.(3, –1), 90]
a a
(A) x = – a (B) x =  (C) x = 0 (D) x 
2 2
2
2. The equation of the common tangent to the curves y = 8x and xy = – 1 is : [IIT-JEE 2002, Scr.(3, –1), 90]
(A) 3y = 9x + 2 (B) y = 2x + 1 (C) 2y = x + 8 (D) y = x + 2
3. (i) The area of the quadrilateral formed by the tangents at the end points of latus rectum to the ellipse
x2 y2
 = 1 is : [IIT-JEE-2003, Scr.(3, –1) /84]
9 5
(A) 27/4 sq. units (B) 9 sq. units (C) 27/2 sq. units (D) 27 sq. units
Conic Section

x2  
(ii) Tangent is drawn to ellipse
27

+ y2 = 1 at 3 3 cos  , sin  where     0 , 2  . Then the value of
 
 such that sum of intercepts on axes made by this tangent is minimum is :
[IIT-JEE-2003, Scr.(3, –1) /84]
   
(A) (B) (C) (D)
3 6 8 4

4. If tangents are drawn to the ellipse x2 + 2y2 = 2, then the locus of the mid-point of the intercept made by
the tangents between the coordinate axes, is - [IIT-JEE-2004, Scr.(3, –1) /84]
1 1 1 1 1 1 1 1
(A) + =1 (B) + =1 (C) + =1 (D) + =1
x2 2y 2 4x2 2y 2 2x 2 4y2 2x 2 y2

5. A parabola has its vertex and focus in the first quadrant and axis along the line y = x. If the distances of
the vertex and focus from the origin are respectively 2 and 2 2 , then an equation of the parabola is
[IIT-JEE 2006, (3, –1), 184]
(A) (x + y)2 = x – y + 2 2
(B) (x – y) = x + y – 2
(C) (x – y)2 = 8(x + y – 2) (D) (x + y)2 = 8(x – y + 2)

 x2
6. STATEMENT - 1 : The curve y = + x + 1 is symmetric with respect to the line x = 1.
2
because
STATEMENT - 2 : A parabola is symmetric about its axis. [IIT-JEE 2007, Paper-2, (3, –1), 81]
(A) Statement - 1 is True, Statement - 2 is True; Statement - 2 is a correct explanation for
Statement - 1
(B) Statement - 1 is True, Statement - 2 is True; Statement - 2 is NOT a correct explanation for
Statement - 1
(C) Statement - 1 is True, Statement - 2 is False
(D) Statement - 1 is False, Statement - 2 is True

Comprehension (7 to 9) [IIT-JEE 2007, Paper-1, (4, –1), 81]

Consider the circle x2 + y2 = 9 and the parabola y2 = 8x. They intersect at P and Q in the first and the
fourth quadrants, respectively. Tangents to the circle at P and Q intersect the x-axis at R and tangents
to the parabola at P and Q intersect the x-axis at S.

7. The ratio of the areas of the triangles PQS and PQR is


(A) 1 : 2 (B) 1 : 2 (C) 1 : 4 (D) 1 : 8

8. The radius of the circumcircle of the triangle PRS is


(A) 5 (B) 3 3 (C) 3 2 (D) 2 3

9. The radius of the incircle of the triangle PQR is


8
(A) 4 (B) 3 (C) (D) 2
3
10. A hyperbola, having the transverse axis of length 2 sin , is confocal with the ellipse 3x2 + 4y2 = 12.
Then its equation is [IIT-JEE-2007, Paper-1(3, –1)/81]
2 2 2 2
(A) x cosec  – y sec  = 1 (B) x2 sec2 – y2 cosec2 = 1
2 2 2 2 2 2 2 2
(C) x sin  – y cos  = 1 (D) x cos  – y sin  = 1
Conic Section

11. Consider the two curves C1 : y2 = 4x, C2 : x2 + y2 – 6x + 1 = 0. Then,


[IIT-JEE 2008, Paper-1, (3, –1), 82]
(A) C1 and C2 touch each other only at one point
(B) C1 and C2 touch each other exactly at two points
(C) C1 and C2 intersect (but do not touch) at exactly two points
(D) C1 and C2 neither intersect nor touch each other
2 2
12. Consider a branch of the hyperbola x – 2y – 2 2 x–4 2y–6=0
[IIT-JEE-2008, Paper-2(3, –1)/81]
with vertex at the point A. Let B be one of the end points of its latus rectum. If C is the focus of the
hyperbola nearest to the point A, then the area of the triangle ABC is
2 3 2 3
(A) 1 – (B) –1 (C) 1 + (D) +1
3 2 3 2

13. Let (x, y) be any point on the parabola y2 = 4x. Let P be the point that divides the line segment from
(0, 0) to (x, y) in the ratio 1 : 3. Then the locus of P is [IIT-JEE 2011, Paper-2, (3, –1), 80]
2
(A) x = y (B) y2 = 2x (C) y2 = x (D) x2 = 2y

x2 y2
14. Let P(6, 3) be a point on the hyperbola 2
  1 . If the normal at the point P intersects the x-axis at
a b2
(9, 0), then the eccentricity of the hyperbola is [IIT-JEE 2011, Paper-2, (3, –1), 80]
5 3
(A) (B) (C) 2 (D) 3
2 2

x2 y 2
15. The ellipse E1 :   1 is inscribed in a rectangle R whose sides are parallel to the coordinate
9 4
axes. Another ellipse E2 passing through the point (0, 4) circumscribes the rectangle R. The eccentricity
of the ellipse E2 is [IIT-JEE 2012, Paper-1, (3, –1), 70]
2 3 1 3
(A) (B) (C) (D)
2 2 2 4
16. If a tangent to a suitable conic (Column 1) is found to be y = x + 8 and its point of contact is (8, 16),
then which of the following options is the only CORRECT combination?
[JEE(Advanced) 2017, Paper-1,(3, –1)/61]
(A) (III ) (i) (P) (B) (I) (ii) (Q) (C) (II) (iv) (R) (D) (III) (ii) (Q)
Conic Section

EXERCISE # 1
PARABOLA
Section (A)
A-1. (4) A-2. (3) A-3. (4) A-4. (2) A-5. (2) A-6. (1) A-7. (1)
A-8. (3) A-9. (3) A-10. (1) A-11. (4) A-12. (2) A-13. (2) A-14. (1)

Section (B)
B-1. (2) B-2. (1) B-3. (1) B-4. (3) B-5. (1) B-6. (1) B-7. (2)
Section-(C)
C-1. (1) C-2. (4) C-3. (4) C-4. (3) C-5. (4) C-6. (3) C-7. (1)
C-8. (3) C-9. (1)
Section-(D)
D-1. (2) D-2. (1) D-3. (4) D-4. (3) D-5. (2) D-6. (4) D-7. (2)
D-8. (2) D-9. (3) D-10. (2) D-11. (2) D-12. (3) D-13. (1) D-14. (2)
D-15. (2)

ELLIPSE
Section (A)
A-1. (2) A-2. (1) A-3. (1) A-4. (4) A-5. (3) A-6. (3) A-7. (4)
A-8. (2) A-9. (4) A-10. (3) A-11. (2) A-12. (2) A-13. (2) A-14. (3)
A-15. (2) A-16. (4) A-17. (3) A-18. (1)

Section (B) :
B-1. (1) B-2. (3) B-3. (2) B-4. (4) B-5. (1) B-6. (3) B-7. (2)
B-8. (3) B-9. (1)

Section (C) :
C-1. (1) C-2. (1) C-3. (2) C-4. (2) C-5. (2) C-6. (1) C-7. (3)
C-8. (3) C-9. (3) C-10. (3) C-11. (2) C-12. (1) C-13. (1)

HYPERBOLA

Section- (A) :
A-1. (2) A-2. (3) A-3. (2) A-4. (3) A-5. (1) A-6. (3) A-7. (1)

A-8. (2) A-9. (3) A-10. (2) A-11. (1) A-12. (2) A-13. (1) A-14. (1)

A-15. (1) A-16. (4) A-17. (1) A-18. (2) A-19. (4)
Section-(B)
B-1. (1) B-2. (1) B-3. (1) B-4. (1) B-5. (4) B-6. (2) B-7. (2)
B-8. (3) B-9. (1) B-10. (2)
Conic Section

Section- (C):

C-1. (4) C-2. (4) C-3. (4) C-4. (1) C-5. (2) C-6. (4)

Section-(D):
D-1. (1) D-2. (1) D-3. (4) D-4. (4) D-5. (4) D-6. (3) D-7. (4)

D-8. (1)

EXERCISE # 2

PART - I

1. (1) 2. (3) 3. (1) 4. (3) 5. (4) 6. (4) 7. (3)


8. (4) 9. (1) 10. (2) 11. (1) 12. (1) 13. (1) 14. (3)
15. (1) 16. (4) 17. (1) 18. 
(3) 19. (3) 20. (3) 21. (2)
22. (1) 23. (1) 24. (1) 25. (4) 26. (3) 27. (1) 28. (4)
29. (4) 30. (4) 31. (4) 32. (3) 33. (1) 34. (1) 35. (3)
36. (3) 37. (1)

PART - II

A-1. (3) A-2. (3) A-3. (3)

Section (B) :
B-1. (3)

Section (C) :
C-1. (1,2,4) C-2. (1,2,3,4) C-3. (2,4) C-4. (3,4)

EXERCISE # 3

PART - I
1. (4) 2. (4) 3. (1) 4. (1) 5. (4) 6. (4) 7. (4)

8. (4) 9. (1) 10. (1,2) 11. (2) 12. (4) 13. (2) 14. (1)

15. (1) 16. (3) 17. (4) 18. (4) 19. (4) 20. (2) 21. (2)

22. (2) 23. (2) 24. (3) 25. (4)

PART - II
1. (C) 2. (D) 3. (i) (D) (ii) (B) 4. (C) 5. (C) 6. (A)

7. (C) 8. (B) 9. (D) 10.(A) 11. (B) 12. (B) 13. (C)

14. (B) 15. (C) 16. (A)


Conic Section

PART - I : PRACTICE TEST PAPER


This Section is not meant for classroom discussion. It is being given to promote self-study and self
testing amongst the Resonance students.

Max. Marks : 120 Max. Time : 1 Hr.

Important Instructions :
1. The test is of 1 hour duration and max. marks 120.
2. The test consists 30 questions, 4 marks each.
3. Only one choice is correct 1 mark will be deducted for incorrect response. No deduction from the total
score will be made if no response is indicated for an item in the answer sheet.
4. There is only one correct response for each question. Filling up more than one response in any
question will be treated as wrong response and marks for wrong response will be deducted accordingly
as per instructions 3 above.

1. The equation of the conic with focus at (1,–1) , directrix along x – y + 1 = 0 and with eccentricity 2 is
(1) x2 – y2 = 1 (2) xy = 1
(3) 2xy – 4x + 4y + 1 = 0 (4) 2xy + 4x – 4y – 1 = 0

2. The length of latus rectum of a parabola whose directrix is x + y – 2 = 0 and focus is (3,–4) is
3 3
(1) 6 2 (2) 3 2 (3) (4)
2 2 2

3. The equation of parabola whose vertices is (–1,–2), axis is vertical and which passes through the point
(3,6) is
(1) x2 + 2x – 2y – 3 = 0 (2) 2x2 = 3y
2
(3) x – 2x – y + 3 = 0 (4) x2 = 3y

4. Equation of tangent to the parabola x2 = 4ay is


2 2 2 2
(1) ty = x + at (2) ty = – x + at (3) tx + y = at (4) tx – y = at

1
5. If slope of the tangent to the parabola y2 = – 8 x is then equation of the tangent is
2
(1) x + 2y + 8 = 0 (2) x – 2y + 8 = 0 (3) x – 2y – 8 = 0 (4) 2x – y – 8 = 0

6. The equation of the tangent to the parabola y2 = 16 x, which is perpendicular to the line y = 3x + 7 is
(1) y – 3x + 4 = 0 (2) 3y – x + 36 = 0 (3) 3y + x – 36 = 0 (4) 3y + x + 36 = 0
2 2
7. Equation of the tangent common to the parabola y = 4x and x = – 32y is
(1) 2x + y – 4 =0 (2) x + 2y – 4 = 0 (3) x – 2y + 4 =0 (4) x – 2y – 4 =0

8. Two perpendicular tangents to y2 = 4ax always intersect on the line-


(1) x = a (2) x + a = 0 (3) x + 2a = 0 (4) x + 4a = 0

a 
9. The equation of the normal at the point  ,a  to the parabola y2 = 4ax , is
4 
(1) 4x + 8y + 9a = 0 (2) 4x + 8y – 9a = 0 (3) 4x + y – a = 0 (4) 4x – y + a = 0
Conic Section

10. Let P be the point on the parabola y2 = 8x which is at a minimum distance from centre C of the circle
2 2
x + (y + 6) = 1 then the equation of the circle having end point of diameter as P and C is
2 2
(1) x + y – 2x – 10y + 24 = 0 (2) x2 + y2 – 4x – 10y – 24 = 0
(3) x2 + y2 + 2x + 10y – 24 = 0 (4) x2 + y2 – 2x + 10y + 24 = 0
11. The locus of the mid point of all chord parallel to the line y = 2x + 3 to the parabola y2 = 8x is
(1) y = 2 (2) y = –2 (3) 2y + 1 = 0 (4) y = 8
12. If the latus rectum of an ellipse be equal to half of its minor axis, then its eccentricity is
1 3 2 2
(1) (2) (3) (4)
2 2 3 3
13. The equation of the ellipse whose one of the vertex is (0,7) and the corresponding directrix is y = 12 is
2 2
(1) 95x + 144y = 4655 (2) 144x2 + 95y2 = 4655
2 2
(3) 144x + 95y = 4750 (4) 95x2 + 144y2 = 13685
14. Equation of the ellipse whose focii are (2,2) and (4,2) and the major axis is of the length 10 is
(1) 25 (x – 3)2 + 24 (y – 2)2 = 600 (2) 25 (x + 3)2 + 24 (y + 2)2 = 600
2 2 2 2
(3) 24 (x – 3) + 25 (y – 2) = 600 (4) 24 (x + 3) + 25 (y + 2) = 600
2 2
15. The equation of the tangent to the ellipse x + 16y = 16 making an angle of 60º with x-axis may be
(1) y = 3x + 47 (2) y = 3x – 13 (3) y = 3x + 16 (4) y = 3x +7

16. Equations of the tangent lines of the ellipse 9x2 + 16y2 = 144 which passes through the point (2,3) is
(1) y = – 3 and x + y = 5 (2) x – y = – 1 and x + y = 5
(3) x – y = – 1 and y = 3 (4) y = 3 and x + y = 5

17. The angle between pair of tangents drawn to the ellipse 3x2 + 2y2 = 5 from the point (1,2) is
 12   12 
(1) tan–1   (2) tan–1   (3) tan–1 (6 5 ) (4) tan–1 (12 5 )
 5   5

18. Equation of the normal at the point (2,3) on the ellipse 9x2 + 16y2 = 180 is
(1) 3y = 8x – 10 (2) 3y = 8x – 7 (3) 3x + 8y – 30 = 0 (4) 3x – 8y + 18 = 0

19. For each point (x,y) on an ellipse , the sum of the distance from (x,y) to the points (2,0) and (–2,0) is 8.
Then the positive value of x so that (,3) lies on the ellipse is -
1
(1) 2 (2) 2 3 (3) (4) 4
3

20. An ellipse is described by using an endless string which is passed over two points. If the axes are 6 cm
and 4 cm, the necessary length of the string and the distance between the pins respectively in cm, are
(1) 6, –2 5 (2) 6 + 2 5 ,2 5 (3) 6,2 + 5 (4) 6, 5

21. The line passing through the extremity A of the major axis and extremity B of the minor axis of the
ellipse
2 2
x + 9y = 9 meets its auxiliary circle at the point M, then the area of the triangle with vertices at A,M
and the origin O is
31 29 21 27
(1) (2) (3) (4)
10 10 10 10
2 2
22. If P is a point on the hyperbola 16x –9y = 144 whose focii are S1 and S2 then |PS1 – PS2| is equal to
(1) 4 (2) 6 (3) 8 (4) 12

23. The locus of the point of intersection of the lines bxt – ayt = ab and bx + ay = abt is
(1) A parabola (2) An ellipse (3) A hyperbola (4) A straight line
Conic Section

24. The equation of the hyperbola in the standard form (with transverse axis along the x-axis) having the
5
length of latus rectum = 9 units and eccentricity = is -
4
x2 y 2 x 2 y2 x2 y2 x2 y2
(1) – 1 (2) – 1 (3) –  1 (4) –  1
64 36 36 64 36 64 64 36

25. The locus of the centre of a circle, which touches externally the given two circle’s is
(1) circle (2) parabola (3) ellipse (4) hyperbola

26. Let 16x2 – 3y2 – 32x – 12y – 44 = 0 be a hyperbola, then which of the following does not hold
32
(1) Length of the transverse axis is 2 3 (2) Length of latus rectum is
3
19 3
(3) Eccentricity is (4) Equation of a directrix is x =
3 19
2 2
x y
27. If PQ is a double ordinate of the hyperbola –  1 such that OPQ is equilateral, O being the
a2 b2
centre, then the eccentricity e satisfies
 2  11 3 2
(1) e   1,  (2) e = (3) e = (4) e >
 3 10 2 3

28. Equation of common tangent to the parabola y2 = 8x and hyperbola 3x2 – y2 = 3 is/are
(1) 2x ± y + 1 = 0 (2) 2x ± y – 1 = 0 (3) x ± 2y + 1 = 0 (4) x ± 2y – 1 = 0

x2 y 2
29. If the two tangents drawn on hyperbola –  1 in such a way that the product of their gradients is
a2 b2
c2, then they intersects on the curve
(1) y2 + b2 = c2 (x2 – a2) (2) y2 + b2 = c2 (x2 + a2)
2 2 2
(3) ax + by = c (4) x2 + y2 = c2 (y2 + b2)
x2 y 2
30. Let C be the centre of the hyperbola –  1 . The tangents at any point P on this hyperbola meets
a2 b2
the straight line bx – ay = 0 and bx + ay = 0 in the point Q and P respectively,then CQ . CR =
1 1 1 1
(1) a2 + b2 (2) a2 – b2 (3)  (4) –
a2 b2 a2 b2
Practice Test (JEE-Main Pattern)
OBJECTIVE RESPONSE SHEET (ORS)

Que. 1 2 3 4 5 6 7 8 9 10

Ans.

Que. 11 12 13 14 15 16 17 18 19 20

Ans.

Que. 21 22 23 24 25 26 27 28 29 30

Ans.
Conic Section

PART - II : PRACTICE QUESTIONS


1. At any point P on the parabola y2 – 2y – 4x + 5 = 0 a tangent is drawn which meets the directrix at Q.
1
The locus of point R, which divides QP externally in the ratio : 1 is
2
(1) (x + 1) (y – 1)2 + 4 = 0 (2) (x + 1) (y – 1)2 + 2 = 0
2
(3) (x + 1) (y – 1) – 4 = 0 (4) (x + 1) (y + 1)2 – 4 = 0

2. Let S be the focus of the parabola y2 = 8x and let PQ be the common chord of the circle x 2 + y2 – 2x –
4y = 0 and the given parabola. The area of the triangle PQS is
(1) 2 (2) 4 (3) 6 (4) 8

3. The equation of the common tangent touching the circle (x – 3)2 + y2 = 9 and the parabola y2 = 4x
above the x-axis is
(1) 3y = 3x + 1 (2) 3y = – (x + 3) (3) 3y =x + 3 (4) 3y = –(3x + 1)

4. The equation of the directrix of the parabola y2 + 4y + 4x + 2 = 0 is:


(1) x =  1 (2) x = 1 (3) x =  3/2 (4) x = 3/2

5. The locus of the mid point of the line segment joining the focus to a moving point on the parabola y2 =
4ax is another parabola with directrix
a a
(1) x = – a (2) x =  (3) x = 0 (4) x 
2 2

6. The equation of the common tangent to the curves y2 = 8x and xy = – 1 is


(1) 3y = 9x + 2 (2) y = 2x + 1 (3) 2y = x + 8 (4) y = x + 2

7. Normals are drawn from the point P with slopes m1, m2, m3 to the parabola y2 = 4x. If locus of P with m1
m2 =  is a part of the parabola itself then =
(1) 0 (2) 1 (3) 2 (4) 3

8. Find the point on x2 + 2 y2 = 6, which is nearest to the line x + y = 7


(1) (2, 1) (2) (–2, 1) (3) (–2, –1) (4) (2, –1)

x2  
9. Tangent is drawn to ellipse
27
 
+ y2 = 1 at 3 3 cos  , sin  where    0 ,  . Then the value of 
 2
such that sum of intercepts on axes made by this tangent is minimum is :
   
(1) (2) (3) (4)
3 6 8 4
2 2
10. The normal at a point P on the ellipse x + 4y = 16 meets the x-axis at Q. If M is the mid point of the
line segment PQ, then the locus of M intersects the latus rectum of the given ellipse at the points
 3 5 2  3 5 19   1  4 3
(1)   ,  (2)   ,  (3)  2 3,   (4)  2 3,  
 2 7   2 4   7  7 
  
A
11*. In a triangle ABC with fixed base BC, the vertex A moves such that cos B + cos C = 4 sin2 . If a, b
2
and c denote the lengths of the sides of the triangle opposite to the angles A, B and C respectively,
then
(1) b + c = 4a (2) b + c = 2a
(3) locus of points A is an ellipse (4) locus of point A is a pair of straight lines
Conic Section

x2 y2
12. The line 2x + y = 1 is tangent to the hyperbola 2
– =1 . If this line passes through the point of
a b2
intersection of the nearest directrix and the x-axis, then the eccentricity of the hyperbola is
(1) 3/2 (2) 2 (3) 5/2 (4) 3

x2 y2
13. Tangents are drawn from any point on the hyperbola – = 1 to the circle x2 + y2 = 9. The locus of
9 4
mid-point of the chord of contact is
(1) x2 + y2 = 5 (2) x2 – y2 = 5
2 2
x2 y 2  x 2 – y 2  x2 y2  x2  y2 
(3)  =  (4)  =  
9 4  9  9 4 
 9 
14. The line passing through the extremity A of the major axis and extremity B of the minor axis of the
ellipse
x2 + 9y2 = 9 meets its auxiliary circle at the point M. Then the area of the triangle with vertices at A, M
and the origin O is
31 29 21 27
(1) (2) (3) (4)
10 10 10 10
15. The common tangents to the circle x2 + y2 = 2 and the parabola y2 = 8x touch the circle at the points P,
Q and the parabola at the points R, S. Then the area of the quadrilateral PQRS is
(1) 3 (2) 6 (3) 9 (4) 15

COMPREHENSION
Comprehension # 1 (Q. 16 to 18)
Normals are drawn at point P, Q and R lying on the parabola y2 = 4x which intersect at (3, 0). Then

16. Area of PQR is


(1) 2 (2) 3 (3) 5 (4) 5/2

17. Centroid of PQR is


(1) (2, 0) (2) (2/3, 0) (3) (5/2, 0) (4) (0, 5/2)

18. Circumcentre of PQR is


(1) (2, 0) (2) (2/3, 0) (3) (5/2, 0) (4) (0, 5/2)
Comprehension # 2 (Q.19 to 21)
Let ABCD be a square of side length 2 units. C2 is the circle through vertices A, B, C, D and C1 is the
circle touching all the sides of the square ABCD. L is a line through A.

PA 2  PB2  PC2  PD2


19. If P is a point on C1 and Q is another point on C2 then is equal to
QA 2  QB2  QC2  QD2
(1) 0.75 (2) 1.25 (3) 1 (4) 0.5

20. A circle touch the line L and the circle C1 externally such that both the circles are on the same side of
the line, then the locus of centre of the circle is
(1) ellipse (2) hyperbola (3) parabola (4) parts of straight line
21. A line M through A is drawn parallel to BD. Point S moves such that its distances from the line BD and
the vertex A are equal. If locus of S cuts M at T2 and T3 and AC at T1, then area of T1T2T3 is
1 2
(1) sq. units (2) sq. units (3) 1 sq. units (4) 2 sq. units
2 3
Conic Section

Comprehension # 3 (22-24)
x2 y 2
Tangents are drawn from the point P(3, 4) to the ellipse  = 1 touching the ellipse at point A and B.
9 4
22. The coordinates of A and B are
 8 2 161   9 8
(1) (3, 0) and (0, 2) (2)  – ,  and – 5, 5
 5 15   

 8 2 161   9 8
(3)  – ,  and (0, 2) (4) (3, 0) and  – , 
 5 15   5 5

23. The orthocentre of the triangle PAB is
 8  7 25   11 8   8 7
(1)  5,  (2)  ,  (3)  ,  (4)  , 
 7 5 8   5 5  25 5 
24. The equation of the locus of the point whose distances from the point P and the line AB are equal, is
2 2
(1) 9x + y – 6xy – 54x – 62y + 241 = 0 (2) x2 + 9y2 + 6xy – 54x + 62y – 241 = 0
2 2
(3) 9x + 9y – 6xy – 54x – 62y – 241 = 0 (4) x2 + y2 – 2xy + 27x + 31y – 120 = 0
Comprehension # 4 (25-26)
x2 y2
The circle x2 + y2 – 8x = 0 and hyperbola – = 1 intersect at the points A and B.
9 4
25. Equation of a common tangent with positive slope to the circle as well as to the hyperbola is
(1) 2x – 5 y – 20 = 0 (2) 2x – 5 y + 4 = 0
(3) 3x – 4y + 8 = 0 (4) 4x – 3y + 4 = 0
26. Equation of the circle with AB as its diameter is
(1) x2 + y2 – 12x + 24 = 0 (2) x2 + y2 + 12x + 24 = 0
(3) x2 + y2 + 24x – 12 = 0 (4) x2 + y2 – 24x – 12 = 0

Comprehension # 5 (27-28)
Let PQ be a focal chord of the parabola y2 = 4ax. The tangents to the parabola at P and Q meet at a
point lying on the line y = 2x + a, a > 0.
27. Length of chord PQ is
(1) 7a (2) 5a (3) 2a (4) 3a
28. If chord PQ subtends an angle  at the vertex of y2 = 4ax, then tan =
2 –2 2 –2
(1) 7 (2) 7 (3) 5 (4) 5
3 3 3 3
Comprehension # 6 (29-30)
2 2 2
Let a, r, s, t be nonzero real numbers. Let P(at , 2at), Q, R (ar , 2ar) and (as , 2as) be distinct points on
2
the parabola y = 4ax. Suppose that PQ is the focal chord and lines QR and PK are parallel, where K is
the point (2a, 0)
29. The value of r is
1 t2  1 1 t2 – 1
(1) – (2) (3) (4)
t t t t

30. If st = 1, then the tangent at P and the normal at S to the parabola meet at a point whose ordinate is
(t 2  1)2 a(t 2  1)2 a(t 2  1)2 a(t 2  2)2
(1) (2) (3) (4)
2t 3 2t 3 t3 t3
Conic Section

Comprehension # 7 (31-33)
y = x is tangent to the parabola y = ax2 + c.
31. If a = 2, then the value of c is
1 1 1
(1) (2) – (3) (4) 1
8 2 2
32. If (1, 1) is point of contact then a is
1 1 1 1
(1) (2) (3) (4)
2 3 4 6
33. If c = 2, then point of contact is
(1) (2, 2) (2) (4, 4) (3) (6, 6) (4) (3, 3)

Comprehension # 8 (34-36)
(3x  4y  10)2 (4x  3y  15)2
+ = 1 is an ellipse
2 3
34. Major and minor axes of the ellipse are
2 3 2 2
(1) 6 and 4 (2) 150 and 100 (3) 10 3 and 10 2 (4) and
5 5
35. Eccentricity of the ellipse is
1 3 2
(1) (2) (3) (4) none of these
3 5 5
36. Centre of the ellipse is
 6 17   2 3
(1) (0, 0) (2)  ,  (3)  , (4) none of these
5 5   5 5 
 
Comprehension # 9 (37-38)
The general second degree equation represent a hyperbola if h2 – ab > 0 and   0 where
a h g
 = h b f . Let H and H be two hyperbolas. They are said to be conjugate to one another if the
g f c
transverse and conjugate axes of one are respectively the conjugate and transverse axes of the other.
Now answer the following questions.
x2 y2
37. Let P be any point on the hyperbola – =1 and PN be the perpendicular on transverse axis.
a2 b2
PN2
Let A and A be the vertices of hyperbola, then =
NA . NA 

a2 b2
(1) (2) (3) a2 (4) b2
b2 a2
38. The director circle of the hyperbola x2 – y2 = a2 is
a2
(1) x2 + y2 = ax2 (2) x2 + y2 = (3) x2 + y2 = a2 (4) none of these
2
Conic Section

PART - I

1. (3) 2. (2) 3. (1) 4. (4) 5. (3) 6. (4) 7. (3)


8. (2) 9. (2) 10. (4) 11. (1) 12. (2) 13. (2) 14. (3)
15. (4) 16. (4) 17. (2) 18. (2) 19. (1) 20. (2) 21. (4)
22. (2) 23. (3) 24. (1) 25. (4) 26. (4) 27. (4) 28. (1)
29. (1) 30. (1)

PART - II

1. (1) 2. (2) 3. (3) 4. (4) 5. (3) 6. (4) 7. (3)


8. (1) 9. (2) 10. (3) 11. (2,3) 12. (2) 13. (4) 14. (4)
15. (4) 16. (1) 17. (2) 18. (3) 19. (1) 20. (3) 21. (3)
22. (4) 23. (3) 24. (1) 25. (2) 26. (1) 27. (2) 28. (4)
29. (4) 30. (2) 31. (1) 32. (1) 33. (2) 34. (4) 35. (1)
36. (2) 37. (1) 38. (4)

You might also like